Примеры с логарифмами: Логарифмы примеры решения задач, формулы и теоретический материал

Содержание

Логарифм. Примеры

Логарифмом числа b по основанию a обозначают выражение . Вычислить логарифм значит найти такой степень x (),при котором выполняется равенство

Основные свойства логарифма

Приведенные свойства необходимо знать, поскольку, на их основе решаются практически все задачи и примеры связаны с логарифмами. Остальные экзотических свойств можно вывести путем математических манипуляций с данными формулами

1.
2.
3.
4.
5.
6.
7.
8.
9.
10.
11.
12.
13.
14.
15.

При вычислениях формулы суммы и разности логарифмов (3,4 ) встречаются довольно часто. Остальные несколько сложные, но в ряде задач являются незаменимыми для упрощения сложных выражений и вычисления их значений.

Распространены случаи логарифмов

Одними из распространенных логарифмов такие в которых основание ровное десять, экспоненте или двойке.
Логарифм по основанию десять принято называть десятичным логарифмом и упрощенно обозначать lg(x). 2 (а>0,с>0).

По свойствам 3,5 вычисляем

2.
По свойству разницы логарифмов имеем

3.
Используя свойства 3,5 находим

4. где .

На вид сложное выражение с использованием ряда правил упрощается к виду

——————————————

Нахождение значений логарифмов

Пример 2. Найти х, если

Решение. Для вычисления применим до последнего слагаемого 5 и 13 свойства

Подставляем в запись и скорбим

Поскольку основания равные, то приравниваем выражения

——————————————

Пример 3. Пусть задано значение логарифмов

Вычислить log[a](x), если

Решение: Прологарифмируем переменную, чтобы расписать логарифм через сумму слагаемых

——————————————

На этом знакомство с логарифмами и их свойствами только начинается. Упражняйтесь в вычислениях, обогащайте практические навыки — полученные знания Вам скоро понадобятся для решения логарифмических уравнений. 2}=2\log_3{|b|}\).
Зачем модуль? Заметим, что в левую часть равенства можно подставлять вместо \(b\) все числа \(b\ne 0\). Если в правой части не поставить модуль (т.е. \(\log_3b\)), то вместо \(b\) можно подставлять только \(b>0\). Таким образом, теряется часть возможных значений числа \(b\).

 

2) В формулах (4): \[\log_a{bc}=\log_a{|b|}+\log_a{|c|} \ \ \ \ \ \ \text{и}
\ \ \ \ \ \ \log_a{\dfrac bc}=\log_a{|b|}-\log_a{|c|}\] аналогичная причина: в левую часть равенств можно подставлять как одновременно положительные \(b\) и \(c\), так и одновременно отрицательные (так как произведение двух отрицательных чисел является положительным числом). А вот в правые части, если в них убрать модули, отрицательные \(b\) и \(c\) уже подставлять будет нельзя (так как аргумент логарифма – всегда положительное выражение). Таким образом, не поставив модули, мы значительно сузим возможные значения для \(b\) и \(c\).
Пример:
Если не поставить модули, а записать, например, \(\log_2{bc}=\log_2b+\log_2c\), то значения \(b=-1\) и \(c=-1\) не удовлетворяют равенству. Тогда как с модулями числа \(b\) и \(c\) могут одновременно быть отрицательными.

примеры решения перехода к новому основанию натурального логарифма и таблица или шпаргалка для этого в 10 классе

Сегодня мы поговорим о формулах логарифмов и дадим показательные примеры решения. Ранее мы уже познакомились с понятием логарифма. А также рассмотрели основные свойства и примеры решения.

Формулы логарифмов сами по себе подразумевают шаблоны решения согласно основным свойствам логарифмов. Прежде применять формулы логарифмов для решения напомним для вас, сначала все свойства.

Формулы логарифмов. Логарифмы примеры решения

Теперь на основе этих формул(свойств), покажем примеры решения логарифмов.

Примеры решения логарифмов на основании формул

Логарифм положительного числа b по основанию a (обозначается logab) — это показатель степени, в которую надо возвести a, чтобы получить b, при этом b > 0, a > 0, а 1.

Согласно определения logab = x, что равносильно ax = b, поэтому logaax = x.

Логарифмы, примеры:

log28 = 3, т.к. 23 = 8

log749 = 2, т.к. 72 = 49

log51/5 = -1, т.к. 5-1 = 1/5

Десятичный логарифм — это обычный логарифм, в основании которого находится 10. Обозначается как lg.

lg100 = 2

log10100 = 2, т.к. 102 = 100

Натуральный логарифм — также обычный логарифм логарифм, но уже с основанием е (е = 2,71828… — иррациональное число). Обозначается как ln.

Формулы или свойства логарифмов желательно запомнить, потому что они понадобятся нам в дальнейшем при решении логарифмов, логарифмических уравнений и неравенств. Давайте еще раз отработаем каждую формулу на примерах.

Основное логарифмическое тождество
a logab = b
Пример.
82log83 = (82log83)2 = 32 = 9

Логарифм произведения равен сумме логарифмов loga (bc) = logab + logac
Пример.
log38,1 + log310 = log3 (8,1*10) = log381 = 4

Логарифм частного равен разности логарифмов
loga (b/c) = logab — logac
Пример.
9 log550/9 log52 = 9 log550- log52 = 9 log525 = 9 2 = 81

Свойства степени логарифмируемого числа и основания логарифма

Показатель степени логарифмируемого числа logab m = mlogab

Показатель степени основания логарифма loganb =1/n*logab

loganb m = m/n*logab,

если m = n, получим loganb n = logab

Пример.

log49 = log223 2 = log23

Переход к новому основанию
logab = logcb/logca,

если c = b, получим logbb = 1

тогда logab = 1/logba

Пример.

log0,83*log31,25 = log0,83*log0,81,25/log0,83 = log0,81,25 = log4/55/4 = -1

Как видите, формулы логарифмов не так сложны как кажутся. Теперь рассмотрев примеры решения логарифмов мы можем переходить к логарифмическим уравнениям.

Источник: https://reshit.ru/formuly-logarifmov-logarifmy-primery-resheniya

Логарифм: что это? Все формулы.

Простейшие уравнения и неравенства

Сейчас речь пойдет о трех страшных буквах: l o g.Существовать в нашем бытии они просто так не могут. Обязательно должен быть какой-нибудь индекс — число снизу (основание логарифма) и число после букв (аргумент логарифма).

Прежде, чем мы перейдем к тому, что такое логарифм, решим парочку подводящих примеров.

Чтобы справиться с этим примером, мы проговариваем в голове: какое число нужно дважды (т.к. корень квадратный) умножить само на себя, чтобы получить 81.

А этот пример можно решить по алгоритму (решения показательных уравнений), а можно так же провести разговор с самим собой (главное не вслух, я считаю это нормально, но кого-то вы можете напугать разговором с самим собой): сколько раз нужно число 3 умножить само на себя, чтобы получить 27. Постепенным перемножением мы дойдем до ответа.

Тогда, если дело касается логарифма:

можно сказать так: в какую степень нужно возвести 3 (число снизу — основание логарифма), чтобы получить 27 (число слева — аргумент логарифма). Не напоминает выше стоящий пример?

На самом деле в этом и заключается основная формула (определение логарифма):

Логарифм говорит нам (кому-то кричит): логарифм числа «b» по основанию «a» равняется числу «c». Тогда без логарифма это можно сформулировать так: чтобы получить число «b», требуется число «a» возвести в степень «c». Логарифм — это действие, обратное возведению в степень.

У отца log есть два родных сына: ln и lg. Так же, как сыновья отличаются возрастом (мы говорим о максимальной точности), так и эти логарифмы отличаются основанием (числовым индексом снизу).

Данные логарифмы придумали для упрощения записи. На самом деле в прикладной математики именно логарифмы по такому основанию встречаются чаще всех остальных. А мы все в глубине души народ ленивый, так что почему бы себе жизнь не упростить?

Что нужно запомнить: ln — это обычный логарифм только по основанию e ( e — это число Эйлера, e = 2,7182…, мой номер телефона, кстати, — это последние 11 цифр числа Эйлера, так что буду ждать звонка).

А lg — это обычный логарифм по основанию 10 (10ая система — это система счисления, в которой мы живем, столько пальцев на руках у среднего человека. В общем 10 — это как 9, только на 1 больше).

Как мы не можем существовать без еды, воды, интернета…  Так и логарифм не представляет свое существование без ОДЗ.

Всегда, когда существует логарифм, должно быть:

«Почему это так?» — это первый вопрос, который я предоставляю тебе. Советую начать с того, что логарифм — это обратное действие от возведения в степень.

А теперь  разберем теорию на практике:

В какую степень нужно возвести два (число в основании), чтобы получить шестнадцать (аргумент логарифма).

Два нужно четыре раза умножить само на себя, чтобы получить 16.

Ответ: 4.

Источник: https://ik-study. ru/ege_math/logharifmy

Логарифмы: правила, основные свойства и формулы :

Логарифмы и правила действий с ними достаточно емкие и простые. Следовательно, разобраться в данной теме вам не составит труда. После того как вы узнаете все правила натуральных логарифмов, любая задача решится самостоятельно.

Первое знакомство с этой темой может показаться скучным и бессмысленным, но именно при помощи логарифмов решились многие проблемы математиков XVI века. «О чем это?» — подумали вы.

Прочтите статью до конца и узнаете, что этот раздел «царицы наук» может быть интересен не только математикам, ученым точных наук, но и простым ученикам средних школ.

Определение логарифма

Начнем с определения логарифма. Как гласят многие учебники: логарифмом числа b по основанию a (logab) является некое число с, для которого выполняется такое равенство: b=ac.

То есть, говоря простыми словами, логарифм — определенная степень, в которую возводим основание, чтобы получить данное число. Но важно помнить, что логарифм вида logab имеет смысл только при: a>0; a — число, отличное от 1; b>0, следовательно, делаем вывод, что логарифм можно найти только у положительных чисел.

Классификация логарифмов по основанию

Логарифмы могут быть с любым положительным числом в основании. Но также существует два вида: натуральный и десятичный логарифмы:

  • Натуральный логарифм — логарифм с основанием е (е — число Эйлера, численно приблизительно равняется 2,7, иррациональное число, которое ввели для показательной функции y = ex), обозначается как ln a = logea;
  • Десятичный логарифм — логарифм с основанием 10, то есть log10a = lg a.

Основные правила логарифмов

Для начала нужно познакомиться с основным логарифмическим тождеством: alogab=b, далее следуют два таких основных правила:

  • loga1 = 0 — так как любое число в нулевой степени равно 1;
  • logaa = 1.

Благодаря открытию логарифма для нас не составит труда решить абсолютно любое показательно уравнение, ответ которого нельзя выразить натуральным числом, а только иррациональным. Например: 5х = 9, х = log59 (так как натурального х для данного уравнения не существует).

Действия с логарифмами:

  • loga(x · y) = logax+ logay — чтобы найти логарифм произведения, нужно сложить логарифмы сомножителей. Обратите внимание на то, что основания логарифмов одинаковы. Если записать это в обратном порядке, то получим правило сложения логарифмов.
  • loga xy = logax — logay — чтобы найти логарифм частного, нужно найти разность логарифмов делимого и делителя. Обратите внимание: основания у логарифмов одинаковы. При записи в обратном порядке получаем правило вычитания логарифмов.
  • logakxp = (p/k)*logax — таким образом, если в аргументе и основании логарифма стоят степени, то их можно выносить за знак логарифма.
  • logax = logac xc — частный случай предыдущего правила, когда показатели степеней равны, их можно сократить.
  • logax = (logbx)(logba) — так называемый модуль перехода, процедура приведения логарифма к другому основанию.
  • logax = 1/logxa — частный случай перехода, смена мест основания и данного числа. Все выражение, образно говоря, переворачивается, и логарифм с новым основанием оказывается в знаменателе.

История возникновения логарифмов

В XVI веке возникла необходимость проведения многих приближенных вычислений для решения практических задач, главным образом, в астрономии (например, определение положения судна по Солнцу или звездам).

Эта потребность быстро росла и значительную трудность создавало умножение и деление многозначных чисел. И ученый-математик Непер при тригонометрических расчетах решил заменить трудоемкое умножение на обыкновенное сложение, сопоставив для этого некоторые прогрессии.

Тогда деление, аналогично, заменяется на процедуру попроще и надежнее — вычитание, а дабы извлечь корень n-ой степени, нужно разделить логарифм подкоренного выражения на n. Решение такой нелегкой задачи в математике явно отображало цели Непера в науке.

Вот как он писал об этом в начале своей книги «Рабдология»:

Я всегда старался, насколько позволяли мои силы и способности, освободить людей от трудности и скуки вычислений, докучливость которых обыкновенно отпугивает очень многих от изучения математики.

Название логарифма предложил сам Непер, он был получен путем совмещения греческих слов, которые в сочетании означали “число отношений”.

Основание логарифма ввел Спейдел. Его заимствовал Эйлер из теории о степенях и перенес в теорию логарифмов. Понятие логарифмирования стало известным благодаря Коппе в XIX веке. А использование натуральных и десятичных логарифмов, а также их обозначения появились благодаря Коши.

В 1614 году Джон Непер издал на латыни сочинение «Описание удивительной таблица логарифмов». Там было изложено краткое описание логарифмов, правил и их свойств. Так термин «логарифм» утвердился в точных науках.

Операцию логарифмирования и первое упоминание о ней появилось благодаря Валлису и Иоганну Бернулли, а окончательно установлена она была Эйлером в XVIII веке.

Именно заслуга Эйлера в распространении логарифмической функции вида y = logax на комплексную область. В первой половине XVIII века вышла его книга «Введение в анализ бесконечных», где были современные определения показательной и логарифмической функций.

Логарифмическая функция

Функция вида y = logах (имеет смысл, только если: а > 0, а ≠ 1).

Логарифмическая функция определяется множеством всех положительных чисел, так как запись logах существует только при условии — х > 0;.

Данная функция может принимать абсолютно все значения из множества R (действительных чисел). Так как у всякого действительного числа b есть положительное x, чтобы выполнялось равенство logaх = b, то есть, это уравнение имеет корень — х = аb (следует из того, что logaab= b).

Функция возрастает на промежутке a>0, а убывает на промежутке 01.

Следует помнить, что любые графики логарифмической функции у = logах имеют одну стационарную точку (1;0), так как logа 1 = 0. Это хорошо видно на иллюстрации графика ниже.

Как видим на изображениях, функция не имеет четности или нечетности, не имеет наибольших или наименьших значений, не ограничена сверху или снизу.

Логарифмическая функция y = logаx и показательная функция y = aх, где (а>0, а≠1), взаимно обратные. Это можно видеть на изображении их графиков.

Решение задач с логарифмами

Обычно решение задачи, содержащей логарифмы, основано на преобразовании их в стандартный вид или же направлено на упрощение выражений под знаком логарифма. Или же стоит переводить обычные натуральные числа в логарифмы с нужным основанием, проводить дальнейшие операции по упрощению выражения.

Есть некие тонкости, которые не стоит забывать:

  • При решении неравенств, когда обе части стоят под логарифмами по правилу с одним основанием, не спешите «отбрасывать» знак логарифма. Помните о промежутках монотонности логарифмической функции. Так как, если основание больше 1 (случай, когда функция возрастает) — знак неравенства останется без изменений, но когда основание больше 0 и меньше 1 (случай, когда функция убывает) — знак неравенства изменится на противоположный;
  • Не забывайте определения логарифма: logах = b, а>0, а≠1 и х>0, чтобы не потерять корней из-за неучтенной области допустимых значений. ОДЗ (область допустимых значений) существует практически для всех сложных функций.

При решении логарифмических уравнений рекомендуется пользоваться равносильными преобразованиями. Также, необходимо быть внимательным и учитывать возможные преобразования, которые способны привести к потере некоторых корней.

Это банальные, но масштабные ошибки, с которыми столкнулись многие на пути поиска верного ответа для задания. Правил решения логарифмов не так уж и много, поэтому эта тема проще, чем другие и последующие, но в ней стоит хорошо разобраться.

Вывод

Данная тема с первого взгляда может показаться сложной и громоздкой, но, исследуя ее глубже и глубже, начинаешь понимать, что тема просто заканчивается, а сложностей так ничего и не вызвало. Мы рассмотрели все свойства, правила и даже ошибки, касающиеся темы логарифмов. Успехов в обучении!

Источник: https://www.syl.ru/article/407401/logarifmyi-pravila-osnovnyie-svoystva-i-formulyi

Свойства логарифмов

Логарифмом положительного числа b по основанию a (a>0, a не равно 1) называют такое число с, что ac = b: log a b=c⇔ a c =b (a>0,a≠1,b>0)

Обратите внимание: логарифм от неположительного числа не определен. Кроме того, в основании логарифма должно быть положительное число, не равное 1. Например, если мы возведем -2 в квадрат, получим число 4, но это не означает, что логарифм по основанию -2 от 4 равен 2.

Основное логарифмическое тождество

a log a b =b (a>0,a≠1) (2)

Важно, что области определения правой и левой частей этой формулы отличаются. Левая часть определена только при b>0, a>0 и a ≠ 1. Правая часть определена при любом b, а от a вообще не зависит. Таким образом, применение основного логарифмического «тождества» при решении уравнений и неравенств может привести к изменению ОДЗ.

Два очевидных следствия определения логарифма

log a a=1 (a>0,a≠1) (3) log a 1=0 (a>0,a≠1) (4)

Действительно, при возведении числа a в первую степень мы получим то же самое число, а при возведении в нулевую степень — единицу.

Логарифм произведения и логарифм частного

log a (bc)= log a b+ log a c (a>0,a≠1,b>0,c>0) (5)

log a b c = log a b− log a c (a>0,a≠1,b>0,c>0) (6)

Хотелось бы предостеречь школьников от бездумного применения данных формул при решении логарифмических уравнений и неравенств. При их использовании «слева направо» происходит сужение ОДЗ, а при переходе от суммы или разности логарифмов к логарифму произведения или частного — расширение ОДЗ.

Действительно, выражение log a (f(x)g(x)) определено в двух случаях: когда обе функции строго положительны либо когда f(x) и g(x) обе меньше нуля.

Преобразуя данное выражение в сумму log a f(x)+ log a g(x) , мы вынуждены ограничиваться только случаем, когда f(x)>0 и g(x)>0. Налицо сужение области допустимых значений, а это категорически недопустимо, т. к. может привести к потере решений. Аналогичная проблема существует и для формулы (6).

Степень можно выносить за знак логарифма

log a b p =p log a b (a>0,a≠1,b>0) (7)

И вновь хотелось бы призвать к аккуратности. Рассмотрим следующий пример:

log a (f (x) 2 =2 log a f(x)

Левая часть равенства определена, очевидно, при всех значениях f(х), кроме нуля. Правая часть — только при f(x)>0! Вынося степень из логарифма, мы вновь сужаем ОДЗ. Обратная процедура приводит к расширению области допустимых значений. Все эти замечания относятся не только к степени 2, но и к любой четной степени.

Формула перехода к новому основанию

log a b= log c b log c a (a>0,a≠1,b>0,c>0,c≠1) (8)

Тот редкий случай, когда ОДЗ не изменяется при преобразовании. Если вы разумно выбрали основание с (положительное и не равное 1), формула перехода к новому основанию является абсолютно безопасной.

Если в качестве нового основания с выбрать число b, получим важный частный случай формулы (8):

log a b= 1 log b a (a>0,a≠1,b>0,b≠1) (9)

Десятичным логарифмом числа x называется логарифм по основанию 10. Десятичные логарифмы используются довольно часто, поэтому для них введено специальное обозначение: log10x = lg x. Все перечисленные выше формулы сохраняют актуальность для десятичных логарифмов. Например, lg(xy)=lgx+lgy (x>0,y>0) .

Натуральным логарифмом числа x (обозначение lnx) называется логарифм х по основанию e. Число e — иррациональное, приближенно равно 2,71. Например, ln e = 1. Пользуясь формулой (8), можно любой логарифм свести к десятичным или натуральным логарифмам: log a b= lgb lga = lnb lna (a>0,a≠1,b>0)

Несколько простых примеров с логарифмами

Пример 1. Вычислите: lg2 + lg50. Решение. lg2 + lg50 = lg100 = 2. Мы воспользовались формулой суммы логарифмов (5) и определением десятичного логарифма.

Пример 2. Вычислите: lg125/lg5. Решение. lg125/lg5 = log5125 = 3. Мы использовали формулу перехода к новому основанию (8).

a log a b =b (a>0,a≠1)
log a a=1 (a>0,a≠1)
log a 1=0 (a>0,a≠1)
log a (bc)= log a b+ log a c (a>0,a≠1,b>0,c>0)
log a b c = log a b− log a c (a>0,a≠1,b>0,c>0)
log a b p =p log a b (a>0,a≠1,b>0)
log a b= log c b log c a (a>0,a≠1,b>0,c>0,c≠1)
log a b= 1 log b a (a>0,a≠1,b>0,b≠1)

Источник: http://www. repetitor2000.ru/svoistva_logarifmov_01.html

Преобразование выражений с использованием свойств логарифмов: примеры, решения

Если у нас есть выражение, содержащее логарифмы, то мы можем преобразовать его с учетом свойств этих логарифмов. В этом материале мы рассмотрим основные правила, по которым осуществляется данное преобразование.

В первом пункте приведем основные свойства логарифмов, представив их в виде списка. Далее рассмотрим характерные примеры преобразований с использованием этих свойств. Отдельно остановимся на числовых выражениях и на выражениях с переменными, а также посмотрим, как преобразовывать примеры с использованием модуля.

Свойства логарифмов

Чтобы преобразовывать выражения с логарифмами, обычно используют выражение, называемое основным логарифмическим тождеством: alogab=b, a>0, a≠1, b>0. Также нужно помнить следующие свойства:

Определение 1

  1.    loga1=0 при любом a>0, a≠1.   
  2. logaa=1, если a>0, a≠1.   
  3.  logaa=1logaa=1 при любом a>0, a≠1.
  4. logaa=1, если a>0, a≠1.
  5. logaap=p, при этом a>0, a≠1 и p может быть любым действительным числом.
  6.  loga(x·y)=logax+logay, a>0, a≠1, x>0, y>0. В обобщенном виде это свойство можно представить как loga(x1·x2·…·xn)=logax1+logax2+…+logaxn, a>0, a≠1, x1>0, x2>0, …, xn>0
  7. ) logaxy=logax-logay.
  8.  logaxy=logax-logay , при этом a>0, a≠1, x>0, y>0.
  9.  logaxy=logax-logay , a>0, a≠1, x>0, y>0.
  10.  logabp=p·logab, при этом a>0, a≠1, b>0, а p может быть любым действительным числом.
  11. это свойство является следствием предыдущего: logabn=1n·logab , a>0, a≠1, n может быть любым натуральным числом больше 1, b>0. 
  12. logab=logcblogca , при этом a>0, a≠1, b>0, c>0, c≠1.
  13. свойство, также являющееся следствием: logab=1logba , где a>0, a≠1, b>0, b≠1.
  14. logaqbp=pq·logab , a>0, a≠1, b>0, p и q могут быть любыми действительными числами, q≠0
  15. logaqbp=pq , a>0, a≠1, p и q – любые действительные числа, q≠0.
  16. logaq ap=pq, blogac=clogab , при этом a>0, a≠1, b>0, c>0.

Преобразовывая выражения, мы можем использовать данные равенства как справа налево, так и наоборот. Учить их все наизусть нет необходимости, достаточно знать основные свойства логарифмов и несколько других свойств, например, что  bn=b1n , если b≥0. Из них можно вывести остальные свойства. Само решение, правда, при этом будет несколько длиннее. Например, если мы не знаем следствия  logaqbp=pq·logab  и используем только основные свойства логарифмов, нам нужно будет выполнить несколько последовательных преобразований:

logaq bp=logabploga aq=p·logabq=pq·logab

То же относится и к последнему свойству из списка, выраженному формулой blogac=Clogac=clogab : оно тоже может быть выведено из основных свойств. Нужно учитывать, что если у нас есть степень положительного числа с логарифмом в показателе, то мы всегда можем поменять число под логарифмом и основание степени местами. В принципе, на практике такие задачи встречаются не слишком часто, но мы их все же разберем.

Как преобразовать числовое выражение с логарифмом

После того, как мы вспомнили основные свойства логарифмов, покажем, как использовать их при решении задач. Начнем с того, как преобразовывать числовые выражения, потому что такие вычисления считаются более простыми. Возьмем сперва примитивные примеры, с помощью которых легко проиллюстрировать выбор нужного свойства логарифма, а потом будем наращивать сложность задач. В конце разберем задания, в которых нужно использовать сразу несколько свойств.

Как выбрать свойство логарифма для преобразования

Список свойств, приведенный в первом пункте, довольно большой, и очевидно, что нужно хорошо в нем ориентироваться, чтобы получить нужный результат. Обычно выбор делается по итогам сравнения исходного логарифма/выражения с левыми и правыми частями формул, выражающих свойства. В том случае, когда одна из частей формулы похожа на исходный логарифм или выражение, мы берем именно это свойство и выполняем преобразование с его помощью. Покажем на примерах, как именно это делается.

Для начала преобразуем выражение, используя определение логарифма, выраженное формулой alogab=b, a>0, a≠1, b>0.

Пример 1

Условие: преобразуйте и вычислите значение следующих выражений: 1) 5log54 ; 2) 10lg(1+2·π), 3) 2+3log2+3 ln15 ; 4) 2log2(−7); 5) (-5)log-5 e3

Решение

В первом примере прослеживается формула alogab. У нас есть a=5, b=4, что соответствует необходимому условию a>0, a≠1, b>0. Используем нужное равенство alogab=b и получим 5log54=4.

Во втором случае a будет равно 10, b – 1+2·π. Необходимое условие выполнено, значит, мы можем записать это в виде равенства: 10lg(1+2·π) =1+2·π.

В третьем выражении у нас есть степень вида alogab, причем a=2+3  и b=ln15. Запишем: 2+3log2+3 ln15=ln15 . Хотя равенство также соответствует формуле alogab, где a равно 2, а b=-7, мы не можем воспользоваться ею для преобразования. Из-за наличия отрицательного числа под знаком логарифма выражение лишается смысла. Кроме того, -7 не соответствует условию b>0, что еще раз подтверждает, что данную формулу мы взять не можем. Следовательно, вычислить значение исходного выражения нельзя, и запись 2log2(−7) =−7 будет ошибочна.

То же самое относится и к четвертому примеру. Мы не можем записать, что -5log-5·e3=e3 , поскольку такое выражение смысла не имеет.

Ответ:1) 5log54=4; 2) 10lg(1+2·π)=1+2·π;  3) 2+3log2+3ln 15=ln 15 ; 4 и 5 — не имеют смысла.

Довольно часто в задачах встречается такой вид преобразования, когда некоторое положительно число представляют в виде степени другого числа, также положительного и не равного 1, имеющего в показателе логарифм. Основной такого преобразования также является основное определение логарифма alogab=b, a>0, a≠1, b>0, но в перевернутом виде, т.е. прочитанное справа налево, например, 3=eln3 или 5=5log55.

Далее возьмем примеры с другими свойствами логарифмов.

Пример 2

Условие: вычислите, если возможно: 1) log−21, 2) log11,3) log01, 4) log71, 5) ln1, 6) lg 1,7) log3,751, 8) log5·π71.

Решение

В первых трех примерах мы видим не имеющие смысла выражения log−21, log11, log01. Основанием логарифма не может быть число меньше 1, в т.ч. 0 и отрицательные значения, т.к. для них логарифм не определен. Значит, значение этих выражений вычислить нельзя.

В других случаях логарифмы имеют подходящие основания: 7, e, 10, 3,75 и 5·π7, а под знаками логарифма везде 1. Зная соответствующее свойство логарифма (loga1=0 при любом a>0, a≠1., мы можем сделать вывод, что значения этих выражений равны 0.

Ответ: 1, 2, 3 смысла не имеют;  4) log71=0, 5) ln1=0,  6) lg1=0,  7) log3,751=0,  8) log5·e71=0.

Пример 3

Условие: вычислите значения: 1) log1313 , 2) ln e, 3) lg 10,4) log5·π3−2(5·π3−2), 5) log−3(−3), 6) log11.

Решение

Нам потребуется свойство логарифма, выраженное формулой logaa=1 при a>0, a≠1. Исходные логарифмы схожи между собой в том, что их основания и числа под знаком логарифма являются одинаковыми. Казалось бы, можно сразу сделать вывод, что значения всех выражений будут равны единице, однако посмотрим внимательнее. В заданиях 1, 2, 3, 4 действительно ответом будет 1, а вот в 5 и 6 исходные выражения смысла не имеют.

Ответ:  1) log13=13=1,  2) ln e=1,  3) lg10=1,  4) log5·π3−2(5·π3−2)=1; 5,6 не имеют смысла.

Пример 4

Условие: вычислите: 1) log3311, 2) log1+22(1+22)723 , 3) logπ5(π5)-2 , 4) log−10(−10)6.

Решение

Видим, что под логарифмами находятся некоторые степени основания, значит, нам нужно использовать соответствующее свойство logaap=p, где a>0, a≠1 и p будет любым действительным числом. С учетом этого можно записать следующее:

  1. log3311=11
  2. log1+22(1+2·2)723=723
  3.  logπ5(π5)-2=-2
  4. для этого примера мы не можем написать такое же равенство, как и в предыдущем примере, поскольку log−10(−10)6=6 не имеет смысла.

Ответ:  1) log3311=11, 2) log1+22(1+2·2)723=723 , 3) logπ5(π5)-2=-2 , 4) не имеет смысла.

Пример 5

Условие: даны выражения log2,64·127, ln2+1π  и lg((−5)·(−12)). Нужно представить их как суммы или разности логарифмов по тому же основанию.

Решение

Смотрим, что находится под знаком логарифма. Там произведение, значит, берем свойство логарифма произведения: loga(x·y) = logax+logay, a>0, a≠1, x>0, y>0. В исходных примерах основания и числа в произведениях положительны, т.е. условие данного свойства соблюдено. Применим его для первого выражения:

log2,64·127=log2,6 4+log2,6127

Чтобы вычислить значение второго выражения, нам нужно свойство логарифма частного: logaxy=logax-logay, a>0, a≠1, x>0, y>0. Здесь в основании стоит положительное число e, также у нас есть положительный числитель 2+1 и знаменатель π, т.е. условия свойства соблюдены. Применяем свойство и записываем, что ln2+1π=ln2+1-ln π .

Разберем третий пример. Начнем с того, что выражение lg((−5)·(−12))  будет иметь смысл, однако формула логарифма произведения для него не подойдет, поскольку оба числа -5 и -12 отрицательны. Значит, преобразование lg((−5) ·(−12))=lg(−5)+lg(−12) не подходит. Какое же свойство тогда использовать?

Проведем предварительное преобразование, чтобы избавиться от отрицательных чисел. Далее мы подробно поговорим, когда нужно выполнять такое действие, а пока ограничимся записью самого решения, которое и так понятно: lg((−5) ·(−12))=lg(5·12)=lg5+lg12.

Ответ:  1) log2,64·127=log2,64+log2,6127 ,  2) ln2+1π=ln2+1-ln π , 3) lg((−5)·(−12))=lg5+lg12.

Пример 6

Условие: упростите выражения log30,25+log316+log30,5 и ln23-ln13 .

Решение

Здесь мы тоже можем использовать свойства логарифма частного и произведения по аналогии с предыдущим примером, только нам потребуется их обратная запись. Преобразуем сумму логарифмов в логарифм произведения, а разность логарифмов в логарифм частного. В итоге у нас получается в первом примере log30,25+log316+log30,5=log3(0,25·16·0,5)=log32, а во втором ln23-ln13=ln23:13=ln 2.

Ответ:  1) log30,25+log316+log30,5=log3(0,25·16·0,5)=log32,  2) ln23-ln13=ln 2 .

Пример 7

Условие: есть выражения log0,7511, log3-1(3-2+5·673)5+1 и log3(−5)6. Нужно избавиться от степени в выражении под знаком логарифма.

Решение

Очевидно, что у нас здесь есть выражения вида logabp. Берем свойство, которое выражается формулой вида

 logabp=p·logab, где a>0, a≠1, b>0, p — любое действительное число. Поскольку условия a>0, a≠1, b>0 выполнены, то мы можем преобразовать logabpв произведение p·logab.

  1.  в случае с первым выражением a равно 7, b – пяти и p– 11. Тогда log0,7511=11·log0,75.
  2.  тут a=3-1, b=3-2+5·673, p=5+1 . Нужные условия выполнены, значит, мы можем записать, что:
    log3-1(3-2+5·673)5+1==5+1·log3-1(3-2+5·673)
  3.  у нас есть выражение той же структуры: logabp, a=3, b=−5, p=6, однако одно из условий не выполняется, а именно b у нас меньше 0. Значит, эту формулу мы применить не можем, и нам будет нужно предварительно преобразовать выражение под знаком логарифма. Решение будет таким: log3(−5)6=log356=6·log35.

Ответ:  1) log0,7511=11·log0,75,  2) log3-1(3-2+5·673)5+1==5+1·log3-1(3-2+5·673) 3) log3(−5)6=6·log35.

Применение формулы в обратном порядке в виде p·logab=logabp требуется довольно часто. При таком преобразовании важно соблюсти все те же условия для числовых значений переменных. Например, 3·ln 5=ln 53 и lg 2·log23=log23lg2.

Пример 8

Условие: согласно таблице логарифмов, lg2≈0,3010 и lg5≈0,6990. Вычислите, сколько будет log25. Здесь же: запишите ln 11ln 3  в виде логарифма, основание которого равно 3.

Решение 

Воспользуемся формулой перехода к новому основанию и представим исходный логарифм как отношение десятичных логарифмов с известными нам значениями.

log25=lg 5lg 2

Вычисляем и находим ответ: lg 5lg 2≈0,69900,3010≈2,3223 .

Во втором примере также будет достаточно формулы перехода к новому основанию, только в обратном порядке, т. е. logcblogca=logab .

Считаем: ln 11ln 3=log311

Ответ: 1) log25≈2,3223,  2) ln 11ln 3=log311 .

Мы разобрали множество примеров, где для осуществления преобразования достаточно применить одну формулу свойства логарифма или его определение. Теперь мы можем перейти к более сложным задачам, в которых нужно последовательно применять несколько свойств, а также делать дополнительные преобразования. Однако перед этим запишем еще один важный пример использования следствий из основных свойств логарифмов.

Пример 9

Условие: 1) дан логарифм ln1+π7 . Необходимо избавиться от корня под знаком логарифма; 2) выполните преобразование дроби 1log25 в логарифм с основанием 4; 3) преобразуйте логарифм loge2345 так, чтобы избавиться от степени в основании; 4) вычислите, сколько будет log2-13216 ; 5) осуществите замену 2,3log73  на степень с основанием 3.

Решение 

  1. Вспоминаем следствие из свойства логарифма степени, которое выражается формулой logabn=1n·logab . В первом случае можем сразу же подсчитать: ln1+π7=17·ln(1+π) .
  2.  во втором случае нам понадобится формула  logab=1logba , примененная в обратном порядке. Получим 1log25=log52.
  3.  здесь нам потребуется свойство logaqbp=pq·logab . Применяем его и получаем loge2345=452·ln 3=25·ln 3.
  4.  в этом случае нам нужно будет следствие, выраженное формулой logaqap=pq: log(2)-13216=16-13=-12
  5.  используем формулу свойства blogac=clogab и вычисляем ответ:

2,3log7·3=3log72,3

Ответ:  1) ln1+π7=17·ln(1+π) ; 2) 1log25=log52 ; 3) loge2345=25·ln 3; 4) log(2)-13216=-12 . 5) 2,3log7·3=3log72,3 .

Задачи с применением нескольких свойств логарифмов

В действительности чаще встречаются более сложные задания, чем те, что мы разобрали в предыдущем параграфе. В них приходится выполнять преобразования в несколько шагов, применяя последовательно одно свойство за другим. Кроме того, они зачастую включают в себя необходимость раскрывать скобки, приводить подобные слагаемые, сокращать обыкновенные дроби и др. Это не так сложно, как кажется, главное – соблюдать правильную последовательность действий.

Пример 10

Условие: вычислите, сколько будет (log315−log35) ·7log75.

Решение

Мы можем заменить выражение в скобках логарифмом log3(15:5), используя свойство частного. Вычисляем его значение и получаем log3(15:5) =log33=1.

Согласно основному определению логарифма, значением 7log75 будет 5. Подставим в исходное выражение получившиеся результаты и найдем, что (log315−log35) ·7log75=1·5=5.

Вот все решение без комментариев:

(log315−log35)·7log75=log3 (15:5)·5==log33·5=1·5=5

Ответ: (log315−log35) ·7log75=5.

Пример 11

Условие: вычислите, чему равен log3log223−1.

Решение

Начнем с преобразования логарифма, который, в свою очередь, сам находится под знаком логарифма. Используем для этого формулу логарифма степени log223=3. Получим, что log3log223=log33, а дальше log33=1. Следовательно, log3log223−1=1−1=0.

Ответ: log3log223−1=0.

Пример 12

Условие: выполните упрощение выражения 3ln 5ln 3log52 .

Решение 

Берем формулу перехода к новому основанию. С ее помощью можно представить отношение логарифмов ln 5ln 3  как log35. У нас получилось 3log35log52 . Теперь применяем формулу основного определения логарифма 3log35=5 и получаем, что 3log35log52 . Нам осталось лишь вычислить значение этого выражения. Оно будет равно 2.

Ответ: 3ln 5ln 3log52=2 .

Перейдем к дальнейшему пункту обсуждения логарифмических преобразований. У нас есть выражения log334, 52+log53,  lg 0,01. Они не напоминают нам ни об одной известной нам формуле свойства, но их все же можно изменить этим способом, если выполнить предварительные преобразования: 52+log53=52·5log53=25·3=75, log334=log31234=412=8 и lg 0,01=lg 10−2=−2. Разберем подробнее, как именно это делается.

Предварительное преобразование перед применением основных свойств логарифмов

На практике мы часто можем встретить логарифмы, которые внешне не похожи ни на одну часть формулы свойства. Однако при этом преобразование требует применения именно этих формул. Это возможно, если перед этим привести их к соответствующему виду. Это процесс называется тождественным преобразованием.

В роли таких действий могут выступать почти любые преобразования выражений, в том числе раскрытие скобок, вынесение за скобки множителей, применение формул тригонометрии и т.д. Это очевидно, поскольку выражения под знаком логарифма могут содержать практически все, что угодно: модули, скобки, дроби, степени и др. Поэтому нужно уметь выполнять разные виды преобразований, чтобы успешно решать такие задачи.

Данная статья не имеет целью осветить все возможные случаи преобразований, поскольку их очень много. Мы выбрали только четыре, которые наиболее распространены.

Определение 2

  1. Довольно часто приходится получать степени под знаком и в основании, чтобы потом использовать формулу логарифма степени и последствия данного свойства. Мы выполняем такое преобразование, если в видим в условии логарифмы следующего вида: ln2532332 , log381.
  2. Также нужно уметь выполнять преобразование, связанное со свойствами степени. Оно нужно нам для последующего использования формулы, которая отвечает определению логарифма. Мы применяем его, когда у нас есть выражения, подобные 2log223, 32·log35, 71+log74, 25(log35)-3  и др.
  3. Обязательно нужно преобразовывать выражения с десятичными дробями под знаком логарифма или в его основании. Иногда после этого мы обнаруживаем, что основание под знаком степени и основание логарифма будут равны, как, например, здесь: log15(0,2)7
  4. Также нужно знать правила преобразования выражений, где под знаком логарифма стоит отрицательное число. Мы расскажем, что нужно делать, если в условии стоят выражения вроде lg(−3)−4, log6((−9)·(−4))  и др.

Разберем подробно каждый вид преобразования.

Как выделить степень в основании логарифма и под его знаком

Сразу возьмем конкретный пример. У нас есть выражение log1981 , структура которого не подсказывает нам ни одного возможного свойства логарифмов, которое можно было бы использовать. Значит, нам надо начать с преобразования самого выражения или сразу с вычисления его значения. Как же это сделать? Обратите внимание на числа 81 и 19. Их легко представить в виде степени с основанием 3 : 81=34 и 19=3−2. Значит, все выражение можно переписать как log3-234 , а здесь уже видна возможность использования свойства logaqbp=pq·logab . Таким образом, log1981=log3-2=4-2=-2 .

Этот пример иллюстрирует нам следующую мысль: если есть возможность, нужно выделить степень в основании и под знаком логарифма, чтобы впоследствии применить свойство логарифма степени. Ниже мы приведем некоторые советы, как именно выделять степени в таких примерах.

В некоторых случаях число можно преобразовать в целую степень, как в примере выше. В задачах то и дело встречаются степени чисел 2 и 3, которые легко узнать с первого взгляда: 243=35, 81=34, 8=23, 64=26  и др. Для решения примеров полезно иметь таблицу степеней натуральных чисел в пределах 10, чтобы сразу видеть возможности преобразования выражений. Также легко работать с выражениями, включающими целые степени 10, 100 и др.

Пример 13

Условие: вычислите или упростите выражения log6216, log3431243, log0,0000010,001.

Решение 

  1. В первом случае мы сразу видим, что 216 можно представить в виде 63. Значит, log6216=log663=3.
  2. у нас есть числа 343 и 1243. Обратимся к таблице степеней и увидим, что их можно представить в виде 73 и 3−4. Выполняем дальнейшие преобразования и получаем:
    log3431243=log733-4==-43·log73=-113·log73
  3.  Поскольку 0,000001=10−6 и 0,001=10−3, тогда log0,0000010,001=log10−610−3= -3-6=12

Ответ: 1) log6216=3,  2) log3431243=-113·log73 ;  3) log0,0000010,001=12.

Если исходного числа нет в таблице степеней, то мы можем разложить его на простые множители.

Пример 14

Условие: упростите выражение log3648·log23.

Решение 

Выполняем разложение 648 на простые множители.

64832416281931222333

Значит, это число можно представить в виде 648=23·34. Следовательно, log3648·log23=log3(23·34)·log23 

Теперь мы можем преобразовать исходный логарифм произведения в сумму, а потом воспользоваться формулой логарифма степени.

log3(23·34)·log23=(log323+log334)·log23= =(3·log32+4)·log23.

Упрощаем выражение через раскрытие скобок:

(3·log32+4)·log23=3·log32·log23+4·log23.

В полученном выражении log32·log23 является произведением взаимно обратных чисел, которое равно 1. Следовательно, формулируем ответ как 3·log32·log23+4·log23=3·1+4·log23=3+4·log23.

Ответ: log3648·log23=3+4·log23.

Зачастую под логарифмом записываются выражения, представляющие собой отношения или произведения корней: 323·3-2, 2·2273  и т.д. Они также приводятся к виду степени: сначала мы выполняем переход от корня к степени, используя соответствующие свойства.  С помощью таких преобразований мы можем получить выражение, удобное для применения формулы логарифма степени.

Нужна помощь преподавателя?

Опиши задание — и наши эксперты тебе помогут!

Описать задание

Пример 15

Условие: найдите значение выражений log52·5-0.5·5-153454  и log372919.

Решение

В первом случае у нас есть произведение степеней, имеющих одинаковые основания. Используя нужное свойство, получим: 52·5−0,5·5−1=52−0,5−1=50,5. Для преобразования дроби сначала выполним переход от корня к степени, затем используем свойство отношения степеней с одинаковыми основаниями:

53454=5344=534-4=5-314

Полученное выражение подставим в исходный логарифм, применив формулу logaqaq=pq, и получим ответ:

log52·5-0.5·5-153454=log50.55-314=-3140.5==-13412=-132=-612

Во втором случае представим число 729 как 36 , а 19 как  3−2. Исходный логарифм приобретет вид log3363-2. Используя свойство корня из степени, преобразуем основание логарифма и получим:

336=333=31-3=3-2

Заканчиваем преобразование: log3363-2=log3-23-2=1 .

Ответ: 1) log52·5-0.5·5-153454=-612 ;  2) log372919=1

Преобразования, которые нужно сделать, чтобы получить под знаком логарифма нужную степень, могут значительно отличаться от примера к примеру.

Пример 16

Условие: вычислите значения log13-1132·3+1-5 и log2·cos1(1+cos2)3 .

Решение

Первое, что нам нужно сделать, – это избавиться от иррациональности в знаменателе первой дроби, лежащей в основании логарифма:

13-1=3+13-1·3+1=3+132-12=3+12

Мы получили результат, схожий с дробью под знаком логарифма. Применим к нему свойства степеней и получим:

132·3+1-5=3+1532=3+1525=3+125

В результате преобразований у нас получился логарифм степени основания log3+123+135 . Значение данного выражения будет равно 5.

Чтобы преобразовать второе выражение, надо воспользоваться тригонометрическими формулами, а конкретно формулой понижения степени cos2a=1+cos2a2 :

log2·cos1(1+cos2)3=log2·cos1(2·cos21)3

Преобразуем второй логарифм, записав его как степень 2·cos2112  или же 2·cos123=2·cos16. Оба выражения будут иметь одно и то же значение, равное шести.

Ответ:  1) log13-1132·3+1-5=5 ;  2) log2·cos1(1+cos2)3=6 .

Как применять свойства степени при преобразовании выражений с логарифмами

Ранее мы уже использовали свойства степеней, чтобы преобразовать выражения под знаком логарифма и его основание. Посмотрим, в каких еще характерных случаях потребуется такая подготовка.

Для начала возьмем задачи на применение свойства степеней с одинаковыми основаниями ap·aq=ap+q. Чаще всего его применяют в обратном порядке, т.е. справа налево.

Пример 17

Условие: вычислите значения 3−2+log37 и 0,72−log0,70,1.

Решение 

В первом примере нужно представить исходную степень как произведение двух степеней, т.е. 3−2+log37=3−2·3log37. Теперь найдем, чему равен первый множитель. Возведем его в степень, потом вычислим значение второго множителя, используя определение логарифма, и подсчитаем их произведение:

3−2·3log37= (19) ·7=79

Во втором примере нам надо подготовить выражение к преобразованию, выполнив переход к произведению степеней: 0,72−log0,70,1=0,72·0,7−log0,70,1.    После этого нам нужно представить показатель −log0,70,1 в виде llog0,7(0,1)−1=log0,710. Теперь все, что нам осталось, – это закончить вычисления:

0,72·0,7−log0,70,1=0,49·0,7log0,710=0,49·10=4,9

Ответ: 1) 3−2+log37=79;  2) 0,72−log0,70,1=4,9.

Также для предварительных преобразований нужно обязательно знать свойство степени в степени, которое выражается формулой (ap)q=ap·q, например, если у нас есть выражение (eln2)3, то мы можем заключить, что значение части в скобках будет равно 2. Значит, (eln2)3=23=8. А если в условии указано (e3)ln2 или (e3)ln2, то мы сначала приводим их к виду (eln2)3:

e3·ln2=eln2·3=(eln2)3 и (e3)ln2=e3·ln2=eln2·3=(eln2)3.

Посмотрим пример решения такой задачи.

Пример 18

Условие: выполните упрощение выражений 2log223−3log23 и 5(log85) −1.

Решение 

Отметим, что выражения 2log223и 2log223 не являются равными друг другу. Мы можем представить 2log223 как   2log23·log23. Используя свойство степени, представим его как (2log23)log23, что будет тождественно равным 3log23. В итоге мы имеем, что 2log223−3log23=3log23−3log23=0.

Вот запись всего решения:

2log223−3log23=2log23·log23−3log23==(2log23)log23−3log23=3log23−3log23=0

Перейдем ко второму примеру. Запись 25(log85)-1 не будет равна (25log85)−1. Мы можем представить степень (log85)−1  как дробь 1log85 . Ее нужно преобразовать, используя следствие свойства перехода к новому основанию по формуле logab=1logba, чтобы получить log58.  

Так, 25(log85)-1=25log58. Поскольку 25 – это 52, имеем 5log58= (52)log58. То, что у нас получилось, представляем в виде (5log58). Нам осталось только вычислить значение: (5log58)2=82=64.

Ответ: 1) 2log223−3log23=0,  2) 25(log85)−1=64.

Также встречаются примеры, где предварительная подготовка к использованию свойств логарифмов заключается в применении и свойства умножения степеней с одинаковыми основаниями, и свойства степени в степени. Например,

4−0,5+2·log43=4−0,5·42·log43==12·(4log43)2=12·32=12·9=4,5

Преобразование логарифмов с десятичными дробями

Применить свойства логарифмов можно и тогда, когда под знаком логарифма у нас стоит десятичная дробь. Что можно сделать с выражением log0,4253? Отметим, что 25 и 0,4 равны между собой 0,4=410=25, то есть это разные формы записи для одного и то же числа.

В целом можно сказать, что в случае наличия десятичной дроби под знаком логарифма необходимо выполнить переход к обыкновенной дроби. Это поможет увидеть возможности использования свойств логарифмов.

Разберем подобную задачу.

Пример 19

Условие: вычислите значение выражения log0,46,25.

Решение 

Начнем с перехода от десятичных дробей к обыкновенным.

log0,46,25=log410625100=log25254

Теперь видно, что мы можем преобразовать 254 в виде (25)−2 и воспользоваться формулой логарифма степени. Вычисляем значение:

 log25254=log2525-2=-2 

Ответ: -2.

Преобразование выражений с отрицательными числами под знаком логарифма

Еще один случай, который мы хотели бы рассмотреть – это преобразование выражений, в которых под знаком логарифма стоит отрицательное число, например, log3-93-27 или log3((−2)·(−5)).

Мы не можем сразу воспользоваться формулами свойств логарифмов в том виде, в каком приводили их в последнем пункте, например, сразу перейти от log3((−2)·(−5)) к log3(−2)+log3(−5), применить свойство логарифма степени к log2(−2)6 или логарифма частного к  log3-93-27 , поскольку отрицательные числа не могут находиться под знаком логарифма и в его основании.

Что это значит на практике? Вернемся к нашему примеру log3((−2)·(−5)). Структура выражения соответствует формуле loga(x·y), где a равно 3, x-3 и y-5. Поскольку условия a>0, a≠1, x>0, y>0 не выполнены, формулу loga(x·y)=logax+logay  мы применить не можем, и равенство log3((−2)·(−5)) =log3(−2) +log3(−5) записать нельзя. Преобразования вида log3-93-27=log3-93-log3(-27) также будут неправильными.

Это не значит, что выражения с отрицательными числами не могут быть преобразованы с использованием свойств логарифмов. Это допускается при условии предварительных преобразований, позволяющих избавиться от минуса. Они базируются на хорошо известных нам правилах работы с числами, меньшими 0.

Вернемся опять к нашему примеру. Согласно правилам умножения, (−2)·(−5) =2·5, значит, log3((−2)·(−5))=log3(2·5). К выражению в таком виде мы уже можем применить формулу log3(2·5) =log32+log35. А вот для примера log2(−2)6 нужно будет выполнить следующие действия:

(−2)6= ((−1)·2)6=(−1)6·26=1·26=26

Значит, log2(−2)6=log226=6.

Пример 20

Условие: найдите значение выражения log2-163-2-23.

Решение

Сначала заключим, что данное выражение имеет смысл. Воспользоваться сразу свойством логарифма частного у нас нет возможности из-за отрицательных чисел под знаком логарифма, поэтому выполним преобразования.

Определив корень нечетной степени из отрицательного числа, выполним переход от -163-2-23  к -163-2-23 . Согласно правилам деления, получим -163-2-23=1632-23 . Теперь нам нужно получившуюся дробь представить в виде степени числа 2 и найти значение получившегося логарифма.

1632-23=2432-23=2432-23=243—23=22log2-163-2-23=log222=2

Ответ: log2-163-2-23=2 .

Некоторые свойства, например, логарифма частного, степени с четным показателем и произведения, можно распространить и на отрицательные числа с помощью модулей. Как это делается, мы покажем далее. Так, поскольку свойство логарифма произведения выглядит как loga(x·y)=loga|x|+loga|y|, где a>0, a≠1, x≠0, y≠0, то после преобразования мы получим log3((−2)·(−5)) =log3|−2|+log3|−5|=log32+log35.  

Как преобразовать логарифмическое выражение с переменными

В предыдущих параграфах мы разобрали, как работать с числовыми выражениями, содержащими логарифмы. Однако если требуется решить логарифмическое неравенство или уравнение, нам понадобится умение работать с теми случаями, когда под знаком логарифма содержится выражение с переменными. В целом при этом мы руководствуемся теми же принципами, что и с числовыми выражениями, но тут следует отдельно пояснить некоторые нюансы, незнание которых ведет к ошибкам.

Особенности преобразований выражений с переменными

Основная трудность состоит в том, что при работе с такими выражениями числа, расположенные под знаком логарифма и в его основании, должны соответствовать особым условиям, а в случае определенных переменных из области допустимых значений эти условия могут оказаться невыполненными. Приведем один наглядный пример.

У нас есть логарифмическое выражение log2(x+1)4. При преобразовании нужно обязательно учитывать область допустимых значений, поэтому первым шагом должно стать ее нахождение. Здесь она определена неравенством (x+1)4>0, значение которого является числовым множеством (−∞, −1) ∪ (−1, +∞). Решить его можно с помощью метода интервалов.

Исходное выражение соответствует формуле logABp, где A равно 2, B – x+1, а p – четырем.

Мы видим, что заданное выражение соответствует виду logABp, где A=2, B=x+1 и p=4. Такие выражения преобразовываются по свойству логарифма степени logabp=p·logab. Можно ли поступить так с этим выражением? Вычислим значение исходного логарифма и выражения, которое получилось после преобразования, например, при x=−2. В итоге: log2(−2+1)4=log21=0, а 4·log2(−2+1) =4·log2(−1) –выражение, не имеющее смысла. Значит, мы ошиблись.

Причина ошибки в том, что мы взяли формулу logabp=p·logab, но это допустимо лишь при условии a>0, a≠1, b>0, p — любое действительное число. Иными словами, проделанное нами преобразование возможно, если x+1>0, что аналогично x>−1 (для A и p – условия выполнены). Однако в нашем случае ОДЗ переменной x для исходного выражения состоит не только из промежутка  x>−1, но и из промежутка x <−1. Но для x <−1 мы не имели права осуществлять преобразование по выбранной формуле.

Почему надо учитывать область допустимых значений

Продолжая работу с выражением log2(x+1)4, проанализируем, как изменится область значений, когда мы выполним переход к виду 4·log2(x+1). Ранее мы уже определили эту область как множество (−∞, −1) ∪ (−1, +∞). Теперь вычислим, какова будет область допустимых значений для 4·log2(x+1). Она определяется условием x+1>0, а ему, в свою очередь, будет отвечать множество (−1, +∞). Мы видим, что область допустимых значений сузилась, а это может привести к различным ошибочным последствиям, поэтому таких преобразований следует избегать.

Важно следить, как меняется область значений во время каждого преобразования. Если на каком-либо этапе происходит ее сужение, это повод тщательно проверить все вычисления и определить, правомерно ли использования данного преобразования.

Чаще всего при решении задач приходится иметь дело с выражениями, область допустимых значений которых не ограничивает применение свойств логарифмов в прямом и обратном порядке, но не следует относиться так ко всем примерам. Нужно всегда проверять, что происходит с областью допустимых значений, и своевременно отслеживать возможные ошибки.

Запишем, в ходе каких преобразований чаще всего происходит непреднамеренное сужение области значений:

Определение 3

  1. когда мы переходим от логарифма произведения к сумме, например, ln(x·(x+3)) =lnx+ln(x+3) сузит нужную область.
  2. Когда мы переходим от логарифма частного к разности. Пример такого преобразования – замена log2xsin x  на log2x−log2sinx .
  3. Когда мы выносим четный показатель степени, используя формулу логарифма степени logabp=p·logab  и формулу logabp=pq·logab . Примеры таких преобразований – logx3(x-8)2=23·logx(x-8) , ln(x+3)−4=−4·ln(x+3).

Иногда в результате преобразования область допустимых значений может не сужаться, а расширяться, например, при переходе от 4·log2(x+1) к log2(x+1)4. В этом случае область расширяется от (−1, +∞) до (−∞, −1) ∪ (−1, +∞).  Такие преобразования имеют место, если оставаться в рамках ОДЗ для исходного выражения. Так, преобразование 4·log2(x+1) =log2(x+1)4 имеет место на области значений переменной x для исходного выражения 4·log2(x+1), то есть, при x+1>0, что аналогично (−1, +∞).

Теперь, когда мы обговорили тонкости, на которые нужно обращать внимание при преобразовании выражений с переменными с использованием свойств логарифмов, остается разобраться, как правильно эти преобразования проводить.

Правила проведения преобразований

Мы говорили ранее, что чаще всего область допустимых значений позволяет нам применять свойства логарифмов в привычных формулировках.

Пример 21

Условие: упростите 3·lg(x+2)7−lg(x+2) −5·lg(x+2)4.

Решение 

На первый взгляд данное выражение нужно преобразовать, используя логарифм степени, то есть сначала вынести нужную степень в виде коэффициента и потом привести подобные слагаемые. Давайте разберемся, правомерно ли применение выбранного свойства в этом случае.

Чтобы перейти от lg(x+2)7 к 7·lg(x+2) и от lg(x+2)4к 4·lg(x+2), нам нужно, чтобы x+2>0. Выясним, будет ли соблюдено данное условие. Для этого нам нужно определить область допустимых значений переменной x. Ее можно выразить с помощью системы неравенств (x+2)7>0,x+2>0,(x+2)4>0 , которая будет равносильной условию x+2>0  (если нужно, повторите материал о решении систем неравенств). Следовательно, мы можем взять формулу логарифма степени. Считаем:

3·lg(x+2)7−lg(x+2)−5·lg(x+2)4==3·7·lg(x+2)−lg(x+2)−5·4·lg(x+2)==21·lg(x+2)−lg(x+2)−20·lg(x+2)==(21−1−20)·lg(x+2)=0

Область допустимых значений позволяет нам использовать и другой вариант вычисления, например, такой:

3·lg(x+2)7-lg(x+2)-5·lg(x+2)4==lg((x+2)7)3-lg(x+2)-lg((x+2)4)5==lg(x+2)21-lg(x+2)-lg(x+2)20==lg(x+2)21(x+2)·(x+2)20=lg1=0

Ответ: 3·lg(x+2)7−lg(x+2) −5·lg(x+2)4=0.

А как быть в случае, если в области допустимых значений нужные условия не будут выполняться? Возьмем соответствующий пример и разберем его.

Пример 22

Условие: выполнить упрощение выражения lg(x+2)4−lg(x+2)2.

Решение

Здесь свободно использовать свойство логарифма степени мы не можем. Область допустимых значений x можно представить в виде объединения промежутков x>−2 и x<−2. Если x>−2, то применяем нужное свойство и действуем по аналогии с тем, как мы решали задачу выше: lg(x+2)4−lg(x+2)2=4·lg(x+2) −2·lg(x+2) =2·lg(x+2). Однако в области значений есть и промежуток x+2<0, и в случае с ним подобное преобразование будет некорректным. Как же нам быть тогда?

Применим знаки модуля. Вспомним определение данного понятия и представим x+2 при x+2<0 как −|x+2|. В таком случае мы можем выполнить переход от lg(x+2)4−lg(x+2)2 к lg(−|x+2|)4−lg(−|x+2|)2, и далее к lg|x+2|4−lg|x+2|2 .То, что у нас получилось в итоге, может быть преобразовано с использованием свойства логарифма степени, ведь |x+2|>0 при любом x.

Модуль нам больше не нужен, значит, избавляемся от него. С учетом того, что мы преобразовывали при |x+2|<0, имеем 2·lg|x+2|=2·lg(−(x+2)). Это и будет ответом на поставленный вопрос.

Ответ: lg(x+2)4-lg(x+2)2=2·lg(x+2), x+2>02·lg(-(x+2)), x+2<0 . Можно записать ответ компактнее, используя знаки модуля: lg(x+2)4-lg(x+2)2=2·lgx+2 .

Возьмем еще один пример, чтобы закрепить навыки работы с модулями.

Пример 23

Условие: представьте выражение lnx-1·x-2x-3  как сумму и разность логарифмов линейных двучленов x−1, x−2 и x−3.

Решение

Вычисляем область допустимых значений данного выражения:

x-1·x-2x-2>0,(1, 2)∪3, +∞

Поскольку значения x−1, x−2 и x−3 будут положительны на промежутке от трех до плюс бесконечности, то мы можем использовать формулы свойств логарифма суммы и разности:

lnx-1·x-2x-3==ln(x-1)+ln(x-2)-ln(x-3)

А на интервале от одного до двух значение x−1 будет положительным, а x−2 и x−3 – отрицательными. Значит, отрицательные значения нам нужно заключить в знаки модуля. У нас получится, что:

lnx-1·x-2x-3=lnx-1·-x-2-x-3==ln(x-1)·x-2x-3

После этого можно спокойно применять формулу логарифма произведений и частного, поскольку на интервале от одного до двух значения всех трех выражений x−1, |x−2| и |x−3| будут положительными. В итоге имеем:

lnx-1·x-2x-3=ln(x-1)+lnx-2-lnx-3==lnx-1+ln(-(x-2))-ln(-(x-3))

Теперь объединяем получившиеся результаты.

Ответ: lnx-1·x-2x-3=lnx-1+ln(-(x-2))-ln(-(x-3)) 

С помощью таких рассуждений и свойств логарифмов отношения, произведения и степени можно вывести несколько результатов, полезных на практике и удобных в использовании:

Определение 4

  1. сумма логарифмов loga|X|+loga|Y|, a>0, a≠1 может быть использована вместо логарифма произведения loga(X·Y).
  2. Разность логарифмов loga|X|−loga|Y|, где a>0, a≠1, X и Y являются произвольными выражениями, может быть использована вместо логарифма частного.
  3. Выражение p·loga|B|, где a>0, a≠1, p является четным числом, а  B – произвольным выражением, может быть использована вместо логарифма B в четной степени p.

Пример 24

Условие: выполните упрощение выражения 13·log8((x+4)·(x-2))-log8x+413x-2 .

Решение

На первый взгляд, мы должны взять формулы логарифмов разности, суммы и степени. Давайте посмотрим, насколько правомерно их использование в данном случае. Для начала вычислим область допустимых значений:

(x+4)·(x-2)>0,(x+4)13x-2>0-∞, -4∪2, +∞

У нас получилось, что значения выражений x+4, x−2 и (x+4)13 в данной области могут быть как положительными, так и отрицательными. Значит, нам нужно использовать модули.

13·log8((x+4)·(x-2))-log8(x+4)13x-2==13·log8x+4+13·log8x-2—log8x+413-log8x-2==13·log8x+4+13·log8x-2—log8(x+14)13+log8x-2==13·log8x+4-log8(x+4)13+14·log8x-2

Зная свойства модуля, перепишем x+413  в виде x+413 . Значит, что:

13·log8x+4-log8(x+4)13+14·log8x-2==13·log8x+4-log8x+413+14·log8x-2

Теперь мы можем свободно применить формулу логарифма степени и выполнить приведение подобных слагаемых:

13·log8x+4-log8x+413+14·log8x-2==13·log8x+4-13·log8x+4+14·log8x-2==14·log8x-2

Возможны и другие преобразования, которые дают тот же результат:

13·log8((x+4)·(x-2))-log8x+413x-2==log8((x+4)·(x-2))13-log8x+413x-2==log8((x+4)13·(x-2))13-log8x+413x-2==log8(x+4)13·(x-2)13x+413x-2=log8(x-2)14

Поскольку на области допустимых значений x−2 может быть и положительным, и отрицательным, необходимо заключить это выражение под знак модуля во время вынесения четного показателя степени. У нас получится, что log8(x-2)14=14·log8x-2

А что было бы, если бы мы не стали использовать модуль, а сразу начали применять свойства логарифмов? У нас получился бы результат 14·log8(x−2), который был бы верен при x∈ (2, +∞), однако ошибочен на всей остальной области допустимых значений.

Ответ: 13·log8((x+4)·(x-2))-log8x+413x-2=14·log8x-2 .

Логарифмы: примеры и решения

Как известно, при перемножении выражений со степенями их показатели всегда складываются (ab*ac = ab+c). Этот математический закон был выведен Архимедом, а позже, в VIII веке, математик Вирасен создал таблицу целых показателей. Именно они послужили для дальнейшего открытия логарифмов. Примеры использования этой функции можно встретить практически везде, где требуется упростить громоздкое умножение на простое сложение. Если вы потратите минут 10 на прочтение этой статьи, мы вам объясним, что такое логарифмы и как с ними работать. Простым и доступным языком.

Определение в математике

Логарифмом называется выражение следующего вида: logab=c, то есть логарифмом любого неотрицательного числа (то есть любого положительного) «b» по его основанию «a» считается степень «c», в которую необходимо возвести основание «a», чтобы в итоге получить значение «b». Разберем логарифм на примерах, допустим, есть выражение log28. Как найти ответ? Очень просто, нужно найти такую степень, чтобы из 2 в искомой степени получить 8. Проделав в уме некоторые расчеты, получаем число 3! И верно, ведь 2 в степени 3 дает в ответе число 8.

Разновидности логарифмов

Для многих учеников и студентов эта тема кажется сложной и непонятной, однако на самом деле логарифмы не так страшны, главное — понять общий их смысл и запомнить их свойста и некоторые правила. Существует три отдельных вида логарифмических выражений:

  1. Натуральный логарифм ln a, где основанием является число Эйлера (e = 2,7).
  2. Десятичный логарифм lg a, где основанием служит число 10.
  3. Логарифм любого числа b по основанию a>1.

Каждый из них решается стандартным способом, включающим в себя упрощение, сокращение и последующее приведение к одному логарифму с помощью логарифмических теорем. Для получения верных значений логарифмов следует запомнить их свойства и очередность действий при их решениях.

Правила и некоторые ограничения

В математике существует несколько правил-ограничений, которые принимаются как аксиома, то есть не подлежат обсуждению и являются истиной. Например, нельзя числа делить на ноль, а еще невозможно извлечь корень четной степени из отрицательных чисел. Логарифмы также имеют свои правила, следуя которым можно с легкостью научиться работать даже с длинными и емкими логарифмическими выражениями:

  • основание «a» всегда должно быть больше нуля, и при этом не быть равным 1, иначе выражение потеряет свой смысл, ведь «1» и «0» в любой степени всегда равны своим значениям;
  • если а > 0, то и аb>0, получается, что и «с» должно быть больше нуля.

Как решать логарифмы?

К примеру, дано задание найти ответ уравнения 10х= 100. Это очень легко, нужно подобрать такую степень, возведя в которую число десять, мы получим 100. Это, конечно же, квадратичная степень! 102=100.

А теперь давайте представим данное выражение в виде логарифмического. Получим log10100 = 2. При решении логарифмов все действия практически сходятся к тому, чтобы найти ту степень, в которую необходимо ввести основание логарифма, чтобы получить заданное число.

Для безошибочного определения значенияя неизвестной степени необходимо научиться работать с таблицей степеней. Выглядит она следующим образом:

Как видите, некоторые показатели степени можно угадать интуитивно, если имеется технический склад ума и знание таблицы умножения. Однако для больших значений потребуется таблица степеней. Ею могут пользоваться даже те, кто совсем ничего не смыслит в сложных математических темах. В левом столбце указаны числа (основание a), верхний ряд чисел — это значение степени c, в которую возводится число a. На пересечении в ячейках определены значения чисел, являющиеся ответом (ac=b). Возьмем, к примеру, самую первую ячейку с числом 10 и возведем ее в квадрат, получим значение 100, которое указано на пересечении двух наших ячеек. Все так просто и легко, что поймет даже самый настоящий гуманитарий!

Уравнения и неравенства

Получается, что при определенных условиях показатель степени — это и есть логарифм. Следовательно, любые математические численные выражения можно записать в виде логарифмического равенства. Например, 34=81 можно записать в виде логарифма числа 81 по основанию 3, равному четырем (log381 = 4). Для отрицательных степеней правила такие же: 2-5= 1/32 запишем в виде логарифма, получим log2 (1/32) = -5. Одной из самых увлекательных разделов математики является тема «логарифмы». Примеры и решения уравнений мы рассмотрим чуть ниже, сразу же после изучения их свойств. А сейчас давайте разберем, как выглядят неравенства и как их отличить от уравнений.

Дано выражение следующего вида: log2(x-1) > 3 — оно является логарифмическим неравенством, так как неизвестное значение «х» находится под знаком логарифма. А также в выражении сравниваются две величины: логарифм искомого числа по основанию два больше, чем число три.

Самое главное отличие между логарифмическими уравнениями и неравенствами заключается в том, что уравнения с логарифмами (пример — логарифм2x = √9)подразумевают в ответе одно или несколько определенных числовых значений, тогда как при решении неравенства определяются как область допустимых значений, так и точки разрыва этой функции. Как следствие, в ответе получается не простое множество отдельных чисел как в ответе уравнения, а а непрерывный ряд или набор чисел.

Основные теоремы о логарифмах

При решении примитивных заданий по нахождению значений логарифма, его свойства можно и не знать. Однако когда речь заходит о логарифмических уравнениях или неравенствах, в первую очередь, необходимо четко понимать и применять на практике все основные свойства логарифмов. С примерами уравнений мы познакомимся позже, давайте сначала разберем каждое свойство более подробно.

  1. Основное тождество выглядит так: аlogaB=B. Оно применяется только при условии, когда а больше 0, не равно единице и B больше нуля.
  2. Логарифм произведения можно представить в следующей формуле: logd(s1*s2) = logds1 + logds2. При этом обязательным условием является: d, s1 и s2 > 0; а≠1. Можно привести доказательство для этой формулы логарифмов, с примерами и решением. Пусть logas1 = f1 и logas2 = f2, тогда af1= s1, af2= s2. Получаем, что s1*s2 = af1*af2= af1+f2 (свойства степеней), а далее по определению: loga(s1*s2)= f1+ f2 = logas1 + logas2, что и требовалось доказать.
  3. Логарифм частного выглядит так: loga(s1/s2) = logas1— logas2.
  4. Теорема в виде формулы приобретает следующий вид: logaqbn = n/q logab.

Называется эта формула «свойством степени логарифма». Она напоминает собой свойства обычных степеней, и неудивительно, ведь вся математика держится на закономерных постулатах. Давайте посмотрим на доказательство.

Пусть logab = t, получается at=b. Если возвести обе части в степень m: atn = bn;

но так как atn= (aq)nt/q = bn, следовательно logaqbn = (n*t)/t, тогда logaqbn = n/q logab. Теорема доказана.

Примеры задач и неравенств

Самые распространенные типы задач на тему логарифмов — примеры уравнений и неравенств. Они встречаются практически во всех задачниках, а также входят в обязательную часть экзаменов по математике. Для поступления в университет или сдачи вступительных испытаний по математике необходимо знать, как правильно решать подобные задания.

К сожалению, единого плана или схемы по решению и определению неизвестного значения логарифма не существует, однако к каждому математическому неравенству или логарифмическому уравнению можно применить определенные правила. Прежде всего следует выяснить, можно ли упростить выражение или привести к общему виду. Упрощать длинные логарифмические выражения можно, если правильно использовать их свойства. Давайте скорее с ними познакомимся.

При решении же логарифмических уравнений, следует определить, какой перед нами вид логарифма: пример выражения может содержать натуральный логарифм или же десятичный.

Вот примеры десятичных логарифмов: ln100, ln1026. Их решение сводится к тому, что нужно определить ту степень, в которой основание 10 будет равно 100 и 1026 соответственно. Для решений же натуральных логарифмов нужно применить логарифмические тождества или же их свойства. Давайте на примерах рассмотрим решение логарифмических задач разного типа.

Как использовать формулы логарифмов: с примерами и решениями

Итак, рассмотрим примеры использования основных теорем о логарифмах.

  1. Свойство логарифма произведения можно применять в заданиях, где необходимо разложить большое значение числа b на более простые сомножители. Например, log24 + log2128 = log2(4*128) = log2512. Ответ равен 9.
  2. log48 = log22 23 = 3/2 log22 = 1,5 — как видите, применяя четвертое свойство степени логарифма, удалось решить на первый взгляд сложное и нерешаемое выражение. Необходимо всего лишь разложить основание на множители и затем вынести значения степени из знака логарифма.

Задания из ЕГЭ

Логарифмы часто встречаются на вступительных экзаменах, особенно много логарифмических задач в ЕГЭ (государственный экзамен для всех выпускников школ). Обычно эти задания присутствуют не только в части А (самая легкая тестовая часть экзамена), но и в части С (самые сложные и объемные задания). Экзамен подразумевает точное и идеальное знание темы «Натуральные логарифмы».

Примеры и решения задач взяты из официальных вариантов ЕГЭ. Давайте посмотрим, как решаются такие задания.

Дано log2(2x-1) = 4. Решение:
перепишем выражение, немного его упростив log2(2x-1) = 22, по определению логарифма получим, что 2x-1 = 24, следовательно 2x = 17; x = 8,5.

Ниже даны несколько рекомендаций, следуя которым можно с легкостью решать все уравнения, содержащие выражения, которые стоят под знаком логарифма.

  • Все логарифмы лучше всего приводить к одному основанию, чтобы решение не было громоздким и запутанным.
  • Все выражение, стоящие под знаком логарифма, указываются как положительные, поэтому при вынесении множителем показателя степени выражения, который стоит под знаком логарифма и в качестве его основания, остающееся под логарифмом выражение должно быть положительно.

Логарифмы примеры с решением и готовыми ответами

Логарифмы

а) Логарифм числа b, где b > 0, по основанию а, где а > 0,

(обозначается ), — показатель степени, в которую нужно возвести число а, чтобы получить число b , т. е.

Это равенство называют основным логарифмическим тождеством.

Логарифм числа а по основанию 10 называют десятичным и обозначают

, а логарифм числа а по основанию е называют натуральным и обозначают .

б) Свойства логарифмов.

Если

то

В частности,

в) Формула перехода к новому основанию.

Если

то

Эта формула называется формулой перехода от логарифма по основанию а к логарифму по основанию с. Частные случаи формулы перехода:

Пример №19.

Освободиться от иррациональности в знаменателе дроби

Решение:

Обозначим

тогда Умножая числитель и знаменатель полученной дроби на и применяя формулу разности кубов, запишем А в следующем виде:

Снова применяя формулу разности кубов, получаем

Ответ:

Пример №20.

Доказать, что

Доказательство. Пусть

Применяя формулу куба суммы и учитывая, что получаем

Таким образом, левая часть А рассматриваемого равенства является корнем уравнения

Это уравнение имеет корень

а его левую часть можно записать в виде Так как уравнение не имеет действительных корней, а левая часть равенства А — действительное число, то

Этот материал взят со страницы решения задач с примерами по всем темам предмета математика:

Решение задач по математике

Возможно вам будут полезны эти страницы:

определение и формулы для умножения частного, примеры решения задач

Что такое логарифм произведения

Определение

Логарифм числа по основанию определяется как показатель степени, в которую надо возвести основание, чтобы получить число.

Число а обычно называют основанием, а число b — аргументом логарифма.

Логарифм имеет следующий вид \(\log_a\left(b\right)\) и читается как «логарифм b по основанию a».

Осторожно! Если преподаватель обнаружит плагиат в работе, не избежать крупных проблем (вплоть до отчисления). Если нет возможности написать самому, закажите тут.

Существуют логарифмы со специальными обозначениями. К ним относятся:

  1. Десятичный логарифм, который обозначается как lg и всегда имеет основание равное 10: \(\log_{10}=lg.\)
  2. Натуральный логарифм, который обозначается как ln и всегда имеет основание равное е = 2,718281… (это число иррационально, его невозможно вычислить точно), \(\log_e=ln.\)

Логарифм также имеет основные свойства, которые необходимо помнить для решения примеров:



 Теорема

Логарифм произведения двух чисел \(x\) и \(y\) равен сумме логарифмов этих чисел, если \(x\), \(y\), \(a\) — положительные числа, где \(a\) — основание и \(a\neq1\).{k+l}\) следует, что \(k+l=\log_a\left(x\times y\right).\)

Если \(k=\log_a\left(x\right)\) и \(l=\log_a\left(y\right)\), то \(\log_a\left(x\times y\right)=\log_a\left(x\right)+\log_a\left(y\right).\)

Логарифм произведения трех положительных чисел

Формула логарифма произведения применяется также и для нескольких положительных множителей. Возьмем для примера три числа и запишем формулу логарифма произведения в преобразованном виде.

\(\log_a\left(x_1\times x_2\times x_3\right)=\log_a\left(x_1\right)+\log_a\left(x_2\right)+\log_a\left(x_3\right)\)

где \(a\) — логарифмическое основание и \(a, x_1,\;x_2,\;x_3 > 0, a\neq0.\)

Логарифм произведения степени, частного

Помимо логарифма произведения, рассмотрим такие понятия как логарифм степени и частного. Они являются не менее важными для решения задач.

Определение

Логарифм степени с положительным основанием — это показатель степени, умноженный на логарифм ее основания.m\right)=m\times\log_a\left(x\right)\)

Определение

Логарифм частного двух положительных чисел — это разность между логарифмом делимого и логарифмом делителя.

\(\log_a\left(\frac xy\right)=\log_a\left(x\right)-\log_a\left(y\right)\)

Примеры решения задач на логарифмы

Задача 1

Вычислить \(\log_{11}\left(21\right).\)

Решение

Разложим аргумент логарифма на более простые числа и применим формулу логарифма произведения. Получим:

\(\log_{11}\left(21\right)=\log_{11}\left(3\times7\right)=\log_{11}\left(3\right)+\log_{11}\left(7\right)\)

Задача 2

Вычислить \(\log_{1,8}\left(1,8\times\sqrt{11}\right).\)

Решение

Применим формулу логарифма произведения для нескольких множителей. Получим:

\(\log_{1,8}\left(1,8\times\sqrt{11}\right)=\log_{1,8}\left(1,8\right)+\log_{1,8}\left(\sqrt{11}\right)=1+\log_{1,8}\left(\sqrt{11}\right)\)

Задача 3

Вычислить \(\log_{12}\left(\frac8{13}\right).2\right)=2\times\log_4\left(5\right)\)

Работа с показателями и логарифмами

Что такое экспонента?

Показатель числа означает , сколько раз
до
использовать
число в умножении.

В этом примере: 2 3 = 2 × 2 × 2 = 8

(2 используется 3 раза при умножении, чтобы получить 8)

Что такое логарифм?

Логарифм идет в обратном направлении.

Возникает вопрос «какой экспонент произвел это?»:

И отвечает на него так:

В этом примере:

  • Экспонента берет 2 и 3 и дает 8 (2, использованное 3 раза в умножении, дает 8)
  • Логарифм берет 2 и 8 и дает 3 (2 дает 8 при трехкратном умножении)

Логарифм говорит , сколько одного числа нужно умножить, чтобы получить другое число

Таким образом, логарифм фактически дает вам показатель степени в качестве ответа :

(Также посмотрите, как связаны экспоненты, корни и логарифмы.)

Работаем вместе

Экспоненты и логарифмы хорошо работают вместе, потому что они «отменяют» друг друга (пока основание «а» одинаково):

Это «обратные функции».

Выполнение одного, затем другого вернет вас к тому, с чего вы начали:

Выполнение a x , затем log a вернет вам снова x :
Выполнение log a , затем a x вернет вам снова x :

Жаль, что они пишутся так иначе … это заставляет вещи выглядеть странно. Таким образом, можно подумать о x как о «верхнем» и записать a (x) как о «нижнем»:

идет вверх, затем вниз, снова возвращает вас обратно: вниз (вверх (x)) = x

идет вниз, затем вверх, снова возвращает вас обратно: вверх (вниз (x)) = x

Во всяком случае, важно то, что:

Логарифмическая функция «отменяется» экспоненциальной функцией.

(и наоборот)

Как в этом примере:

Пример, что такое

x в log 3 (x) = 5

Начать с: log 3 (x) = 5

Мы хотим «отменить» журнал 3 , чтобы получить «x =»

Используйте экспоненциальную функцию (с обеих сторон):
И мы это знаем, так что: x = 3 5

Ответ: x = 243

А также:

Пример: вычислить y в

y = log 4 (1/4)

Начать с: y = log 4 (1/4)

Используйте экспоненциальную функцию с обеих сторон:

Упростить: 4 y = 1/4

Теперь простой трюк: 1/4 = 4 −1

Итак: 4 y = 4 −1

Итак: y = −1

Свойства логарифмов

Одна из сильных сторон логарифмов заключается в том, что они могут превращать в умножение в сложение .

log a (m × n) = log a m + log a n

«Журнал умножения — это сумма журналов»

Почему это правда? См. Сноску.

Используя это свойство и законы экспонент, мы получаем следующие полезные свойства:

бревно a (м × n) = бревно a м + лог a n логарифм умножения — сумма логов
бревно a (м / п) = бревно a м — бревно a n бревно деления разность бревен
журнал a (1 / n) = −log a n это просто следует из предыдущего правила «деления», потому что log a (1) = 0
бревно a r ) = r (бревно a м) логарифм m с показателем r равен r, умноженному на логарифм m

Помните: основание «а» всегда одинаково!

История: Логарифмы были очень полезны до изобретения калькуляторов… например, вместо умножения двух больших чисел, используя логарифмы, вы можете превратить это в сложение (намного проще!)

И в помощь были книги, полные таблиц логарифма.

Давайте повеселимся, используя свойства:

Пример: Упростить

log a ((x 2 +1) 4 √x)

Начать с: log a ((x 2 +1) 4 √x)

Используйте log a (mn) = log a m + log a n : log a ((x 2 +1) 4 ) + log a (√x)

Используйте log a (m r ) = r (log a m) : 4 log a (x 2 +1) + log a (√x)

Также √x = x ½ : 4 журнала a (x 2 +1) + журнал a (x ½ )

Используйте log a (m r ) = r (log a m) снова : 4 log a (x 2 +1) + ½ log a (x)

Это насколько мы можем упростить… мы ничего не можем сделать с логом a (x 2 +1).

Ответ: 4 журнала a (x 2 +1) + ½ журнала a (x)

Примечание: нет правила для обработки журнала a (m + n) или журнала a (m − n)

Мы также можем применить правила логарифма «в обратном порядке», чтобы объединить логарифмы:

Пример: превратите это в один логарифм:

log a (5) + log a (x) log a (2)

Начинается с: log a (5) + log a (x) — log a (2)

Используйте бревно a (мн) = бревно a м + бревно a n : бревно a (5x) — бревно a (2)

Используйте бревно a (м / п) = бревно a м — бревно a n : бревно a (5x / 2)

Ответ: журнал a (5x / 2)

Натуральный логарифм и натуральные экспоненциальные функции

Когда основание — e («Число Эйлера» = 2.718281828459 …) получаем:

  • Натуральный логарифм log e (x) , который чаще записывается ln (x)
  • Естественная экспоненциальная функция e x

И та же идея, что одно может «отменить» другое, все еще верна:

ln (e x ) = x

e (ln x) = x

А вот их графики:

Натуральный логарифм

Естественная экспоненциальная функция

График f (x) = ln (x)

График f (x) = e x

Проходит через (1,0) и (e, 1)

Проходит через (0,1) и (1, e)

Это та же кривая с перевернутыми осями x и y .

Это еще одна вещь, чтобы показать вам, что они являются обратными функциями.

На калькуляторе натуральный логарифм — это кнопка «ln».

Всегда старайтесь использовать натуральные логарифмы и натуральную экспоненциальную функцию, когда это возможно.

Десятичный логарифм

Если база равна 10 , вы получите:

  • Общий логарифм log 10 (x) , который иногда записывается как log (x)

Инженеры любят его использовать, но в математике он мало используется.

На калькуляторе десятичный логарифм — это кнопка «журнал».

Это удобно, потому что показывает, насколько «большое» число в десятичной системе (сколько раз вам нужно использовать 10 при умножении).

Пример: расчет журнала

10 100

Ну, 10 × 10 = 100, поэтому, когда 10 используется 2 раз при умножении, вы получаете 100:

журнал 10 100 = 2

Аналогично log 10 1000 = 3, log 10 10 000 = 4 и т. Д.

Пример: расчет журнала

10 369

Хорошо, лучше всего использовать кнопку «журнал» моего калькулятора:

журнал 10 369 = 2,567 …

Замена базы

Что, если мы хотим изменить основание логарифма?

Легко! Просто используйте эту формулу:

«x поднимается, a понижается»

Или другой способ думать об этом: log b a — это как «коэффициент преобразования» (та же формула, что и выше):

Итак, теперь мы можем преобразовать любую базу в любую другую.

Еще одно полезное свойство:

Видите, как меняются местами «x» и «a»?

Пример: вычислить 1 / лог

8 2

1 / лог 8 2 = лог 2 8

И 2 ​​× 2 × 2 = 8, поэтому, когда 2 используется 3 раз при умножении, вы получаете 8:

1 / лог 8 2 = лог 2 8 = 3

Но мы чаще используем натуральный логарифм, поэтому стоит помнить об этом:

Пример: расчет журнала

4 22

На моем калькуляторе нет кнопки « log 4 »…

… но у него есть кнопка « ln «, поэтому мы можем использовать это:

журнал 4 22 = ln 22 / ln 4

= 3,09 … / 1,39 …

= 2,23 (до 2 знаков после запятой)

Что означает этот ответ? Это означает, что 4 с показателем 2,23 равняется 22. Итак, мы можем проверить этот ответ:

Чек: 4 2,23 = 22,01 (достаточно близко!)

Вот еще пример:

Пример: расчет журнала

5 125

журнал 5 125 = ln 125 / ln 5

= 4.83 … / 1,61 …

= 3 (точно)

Я знаю, что 5 × 5 × 5 = 125, (5 используется 3 раз, чтобы получить 125), поэтому я ожидал ответа 3 , и это сработало!

Использование в реальном мире

Вот несколько примеров использования логарифмов в реальном мире:

Землетрясения

Магнитуда землетрясения в логарифмической шкале.

В знаменитой «шкале Рихтера» используется эта формула:

M = бревно 10 A + B

Где A — амплитуда (в мм), измеренная сейсмографом
, а B — поправочный коэффициент расстояния

В настоящее время есть более сложные формулы, но они все еще используют логарифмическую шкалу.

Звук

Громкость измеряется в децибелах (сокращенно дБ):

Громкость в дБ = 10 log 10 (p × 10 12 )

, где p — звуковое давление.

Кислый или щелочной

Кислотность (или щелочность) измеряется в pH:

pH = −log 10 [H + ]

, где H + — молярная концентрация растворенных ионов водорода.
Примечание: в химии [] означает молярную концентрацию (моль на литр).

Другие примеры

Пример: решить 2 журнала

8 x = журнал 8 16

Начать с: 2 журнала 8 x = журнал 8 16

Поместите в журнал «2»: журнал 8 x 2 = журнал 8 16

Убрать бревна (они же основа): х = 16

Решите: x = −4 или +4

Но … но … но … у вас не может быть журнала с отрицательным числом!

Значит, случай −4 не определен.

Ответ: 4

Проверка: воспользуйтесь калькулятором, чтобы узнать, правильный ли это ответ … также попробуйте случай «−4».

Пример: Решить e

−w = e 2w + 6

Начнем с: e −w = e 2w + 6

Нанесите ln на обе стороны: ln (e −w ) = ln (e 2w + 6 )

и ln (e w ) = w : −w = 2w + 6

Упростить: −3w = 6

Решить: w = 6 / −3 = −2

Ответ: w = −2

Чек: e — (- 2) = e 2 и e 2 (−2) +6 = e 2

Сноска: Почему

журнал (m × n) = log (m) + log (n) ?

Чтобы увидеть , почему , мы будем использовать и:

Сначала превратите m и n в «показатели логарифмов»:

Затем используйте один из законов экспонент

Наконец отмените экспоненты.

Это одна из тех умных вещей, которые мы делаем в математике, которую можно описать как «мы не можем сделать это здесь, поэтому давайте перейдем к там , затем сделаем это, затем вернемся»

Логарифмы

могут иметь десятичные дроби

В разделе «Введение в логарифмы» мы увидели, что логарифм отвечает на такие вопросы:

Сколько двойки мы умножаем, чтобы получить 8?

Ответ: 2 × 2 × 2 = 8 , поэтому нам нужно было умножить 3 из 2 s, чтобы получить 8

Таким образом, логарифм равен 3

И мы пишем «количество двоек, которые мы умножаем, чтобы получить 8, это 3 » как

журнал 2 (8) = 3

Итак, эти две вещи одинаковы:

Пример: что такое журнал

10 (100)…?

10 × 10 = 100

Умножение 2 10 вместе дает 100, поэтому:

журнал 10 (100) = 2

Примечание: используя экспоненты: 10 2 = 100

А теперь зададим новый вопрос:

Пример: Что такое журнал

10 (300) …?

10 × 10 = 100

10 × 10 × 10 = 1000

О нет! Мы либо слишком низки, либо слишком высоки.

Итак, умножения на два 10 недостаточно, но умножение на три 10 — это слишком много…

… а как насчет два с половиной …?

Половина умножения …

Как мы можем умножить на половину ?

Ну, умножить на половину — это то, что нам нужно сделать дважды , чтобы получить целое умножение .

И это квадратный корень!

√10 × √10 = 10

Умножение на квадратный корень похоже на умножение половины.

Итак, давайте попробуем это:

Пример: журнал

10 (300) (продолжение)

Попробуйте умножить 10 на два с половиной :

10 × 10 × √10
=
10 × 10 × 3,16 …
= 316 ….

Мы приближаемся к 300, поэтому можно сказать:

лог 10 (300) ≈ 2,5 (приблизительно)

Другими словами, умножая 10 на два с половиной раза, получаем примерно 300.

(Примечание: используя экспоненты, можно сказать, что 300 ≈ 10 2.5 )

А вот как это выглядит на графике:

2: 10 × 10 = 100
2,5: 10 × 10 × √10 = 316 ….
3: 10 × 10 × 10 = 1000

Итак, логарифмы — это не просто целые числа, такие как 2 или 3: мы нашли значение 2,5 ,

Мы можем найти больше значений (используя кубический корень, корень четвертой степени и т. Д.), Например, 2,75, 1,9055 и т. Д.

Но нам не нужно использовать квадратные корни и т. Д. Для нахождения логарифмов, потому что…

на практике калькулятором пользоваться проще!

Просто используйте калькулятор

Например, кнопка «журнал» отобразит логарифм по основанию 10.

Пример: что такое журнал

10 (300) с помощью калькулятора?

Получите калькулятор, введите 300 , затем нажмите log

Ответ: 2,477…

Это означает, что нам нужно использовать 10 в умножении 2,477 … раз, чтобы получить 300:

журнал 10 (300) = 2,477 …

Наша ранняя оценка 2,5 была не так уж и плоха, не так ли?

Примечание: используя экспоненты: 10 2,477 … = 300

Пример: Что такое журнал

10 (640)?

Возьмите калькулятор, введите 640, затем нажмите log

Ответ: 2.806 …

Это означает, что нам нужно использовать 10 при умножении 2,806 … раз, чтобы получить 640:

журнал 10 (640) = 2,806 …

Посмотрите на график выше и посмотрите, какое значение вы получите при x = 640

Примечание: используя экспоненты: 10 2,806 … = 640

Итак, у вас есть … логарифмы (которые говорят нам, сколько раз использовать число при умножении) могут иметь десятичные значения.

Правила логарифмирования

— объяснение и примеры

Что такое логарифм? Зачем мы их изучаем? А каковы их правила и законы?

Для начала, логарифм числа «b» может быть определен как степень или экспонента, до которой должно быть возведено другое число «a», чтобы получить результат, равный числу b.

Мы можем символически представить это утверждение как;

журнал a b = n.

Точно так же мы можем определить логарифм числа как обратную его экспоненту. Например, log a b = n можно представить экспоненциально как; а н = б.

Следовательно, можно сделать вывод, что;

a n = b ⇔ log a b = n.

Хотя в школах изучают логарифмы для упрощения вычислений с большими числами, они по-прежнему играют важную роль в нашей повседневной жизни.

Давайте посмотрим на некоторые из этих применений логарифмов:

  • Мы используем логарифмы для измерения кислотности и щелочности химических растворов.
  • Измерение силы землетрясений производится по шкале Рихтера с использованием логарифмов.
  • Уровень шума измеряется в дБ (децибелах) по логарифмической шкале.
  • Экспоненциальные процессы, такие как уменьшение соотношения активных изотопов, рост бактерий, распространение эпидемии среди населения и охлаждение мертвого тела, анализируются с использованием логарифмов.
  • Логарифм используется для расчета периода выплаты ссуды.
  • В математике логарифм используется для различения сложных задач и определения площади под кривыми.

Подобно экспонентам, логарифмы имеют правила и законы, которые работают так же, как правила показателей. Важно отметить, что законы и правила логарифмов применяются к логарифмам любого основания. Однако во всех расчетах должна использоваться одна и та же база.

Мы можем использовать законы и правила логарифмов для выполнения следующих операций:

  • Преобразование логарифмических функций в экспоненциальную форму.
  • Сложение
  • Вычитание
  • Умножение
  • Деление
  • Расширение и сжатие
  • Решение логарифмических уравнений.

Законы логарифмов

Логарифмические выражения могут быть записаны по-разному, но в соответствии с определенными законами, называемыми законами логарифмов. Эти законы могут применяться к любой базе, но при расчетах используется одна и та же база.

Четыре основных закона логарифмов включают в себя:

Закон о правилах продукта

Первый закон логарифмов гласит, что сумма двух логарифмов равна произведению логарифмов.Первый закон представлен как;

⟹ журнал A + журнал B = журнал AB

Пример:

  1. журнал 2 5 + журнал 2 4 = журнал 2 (5 × 4) = журнал 2 20
  2. журнал 10 6 + журнал 10 3 = журнал 10 (6 x 3) = журнал 10 18
  • журнал x + журнал y = журнал (x * y) = журнал xy
  1. журнал 4x + log x = log (4x * x) = log 4x 2

Закон о частичном правиле

Вычитание двух логарифмов A и B равно делению логарифмов.

⟹ журнал A — журнал B = журнал (A / B)

Пример:

  1. журнал 10 6 — журнал 10 3 = журнал 10 (6/3) = журнал 10 2
  2. log 2 4x — log 2 x = log 2 (4x / x) = log 2 4

Закон о правиле власти

⟹ log A n = n log A

Пример:

  1. log 10 5 3 = 3 log 10 5
  2. 2 log x = log x 2
  1. 5 ln x 2 = ln x (2 * 5 ) = ln x 10

Изменение закона об основных правилах

⟹ log b x = (log a x) / (log a b)

Пример 4:

  • журнал 4 16 = (журнал 16) / (журнал 4).

Правила логарифмов

Логарифмы — очень дисциплинированная область математики. Они всегда применяются в соответствии с определенными правилами и положениями.

При игре с логарифмами необходимо помнить следующие правила:

  • Учитывая, что a n = b ⇔ log a b = n, логарифм числа b определен только для положительных действительных чисел.

⟹ a> 0 (a ≠ 1), a n > 0.

  • Логарифм положительного действительного числа может быть отрицательным, нулевым или положительным.

Примеры

  1. 3 2 = 9 log 3 9 = 2
  2. 5 4 = 625 log 5 625 = 4
  3. 7 0 = 1 ⇔ log 7 1 = 0
  4. 2 -3 = 1 / 8 ⇔ лог 2 ( 1 / 8 ) = -3
  5. 10 -2 = 0,01 ⇔ лог 10 01 = -2
  6. 2 6 = 64 лог 2 64 = 6
  7. 3 — 4 = 1/3 4 = 1/81 лог 3 1/81 = -4
  8. 10 -2 = 1/100 = 0.01 ⇔ log 10 01 = -2
  • Логарифмические значения данного числа различны для разных оснований.

Примеры

  1. log 9 81 log 3 81
  2. log 2 16 ≠ log 4 16
  • Логарифмы с основанием 10 называются десятичными логарифмами. Когда логарифм записывается без основания индекса, мы предполагаем, что основание равно 10.

Примеры

  1. log 21 = log 10
  2. log 0.05 = log 10 05
  • Логарифм с основанием «е» называется натуральным логарифмом. Константа e приблизительно равна 2,7183. Натуральные логарифмы выражаются как ln x, что совпадает с log e
  • Логарифмическое значение отрицательного числа является мнимым.
  • Логарифм от 1 до любого конечного ненулевого основания равен нулю.
    a 0 = 1 log a 1 = 0.

Пример:

7 0 = 1 ⇔ log 7 1 = 0

  • Логарифм любого положительного числа с тем же основанием равно 1.

a 1 = a ⟹ log a a = 1.

Примеры

  1. log 10 10 = 1
  2. log 2 2 = 1
  • Учитывая, что x = log a M, затем a log a M = a

Пример 1

Вычислите следующее выражение.

журнал 2 8 + журнал 2 4

Решение

Применяя закон правила продукта, мы получаем;

log 2 8 + log 2 4 = log 2 (8 x 4)

= log 2 32

Записываем 32 в экспоненциальной форме, чтобы получить значение его экспоненты.

32 = 2 5

Следовательно, 5 — правильный ответ

Пример 2

Анализ журнала 3 162 — журнал 3 2

Решение

Это выражение вычитания; поэтому мы применяем закон правила частных отношений.

журнал 3 162 — журнал 3 2 = журнал 3 (162/2)

= журнал 3 81

Запишите аргумент в экспоненциальной форме

81 = 3 4

Следовательно, ответ — 4.

Пример 3

Разверните логарифмическое выражение ниже.

журнал 3 (27x 2 y 5 )

Решение

журнал 3 (27x 2 y 5 ) = журнал 3 27 + журнал 3 x 2 + журнал 3 y 5

= журнал 3 (9) + журнал 3 (3) + 2log 3 x + 5log 3 y

Но журнал 3 9 = 3

Заменить, чтобы получить.

= 3 + log 3 (3) + 2log 3 x + 5log 3 y

Пример 4

Вычислить значение log √2 64.

Решение

⟹ log √2 64 = log √2 (2) 6

⟹ log √2 64 = 6log √2 (2)

⟹ log √2 64 = 6log √2 (√2) 2

⟹ log √2 64 = 6 * 2log √2 (√2)

⟹ log √2 64 = 12 * 2 (1)

⟹ log √2 64 = 12

Пример 5

Решите относительно x, если log 0.1 (0,0001) = x

Решение

⟹ log 0,1 (0,0001) = log 0,1 (0,1) 4

⟹ log 0,1 (0,0001) = 4log 0,1 0,1

⟹ log 0,1 (0,0001) = 4 (1)

⟹ log 0,1 (0,0001) = 4

Следовательно, x = 4.

Пример 6

Найдите значение x для данного , 2log x = 4log3

Решение

2logx = 4log3

Разделите каждую сторону на 2.

⟹ log x = (4log3) / 2

⟹ log x = 2log3

⟹ log x = log3 2

⟹ log x = log9

x = 9

Пример 7

Вычислить журнал 2 (5x + 6) = 5

Решение

Перепишите уравнение в экспоненциальной форме

2 5 = 5x + 6

Упростите.

32 = 5x + 6

Вычтем обе части уравнения на 6

32-6 = 5x + 6-6

26 = 5x

x = 26/5

Пример 8

Решить log x + log (x − 1) = log (3x + 12)

Решение

⇒ log [x (x — 1)] = log (3x + 12)

Отбросьте логарифмы, чтобы получить;

⇒ [x (x — 1)] = (3x + 12)

Примените свойство распределения для удаления скобок.

⇒ x 2 — x = 3x + 12

⇒ x 2 — x — 3x — 12 = 0

⇒ x 2 — 4x — 12 = 0

⇒ (x − 6) ( x + 2) = 0

⇒x = — 2, x = 6

Поскольку аргумент логарифма не может быть отрицательным, то правильный ответ — x = 6.

Пример 9

Вычислить ln 32 — ln (2x) = ln 4x

Решение

ln [32 / (2x)] = ln 4x

Отбросьте натуральные бревна.

[32 / (2x)] = 4x

32 / (2x) = 4x.

Перекрестное умножение.

32 = (2x) 4x

32 = 8x 2

Разделите обе стороны на 8, чтобы получить;

x 2 = 4

x = — 2, 2

Поскольку у нас не может быть логарифма отрицательного числа, то x = 2 остается правильным ответом.

Практические вопросы

  1. Оценка журнала 4 64 + журнал 4 16
  2. журнал 3 14−2log 3 5
  3. Оценить 2 журнала 3 5 + log 3 40 — 3 log 3 10
  4. Сжатый журнал 2 4 + log 2 5
  5. Развернуть журнал 3 (xy 3 / √z)
  6. Сжать следующее выражение 5 ln x + 13 ln (x 3 + 5) — 1/2 ln (x + 1)
  7. Упростить журнал a 28 — log a 4 в виде единственного логарифма
  8. Найдите значение журнала 5 8 + 5 (1/1000)
  9. Решите относительно x в логарифме 3log 5 2 = 2log 5 X
  10. Запишите log12 + log 5 как единственный логарифм

Предыдущий урок | Главная страница | Следующий урок

Использование логарифмов в реальном мире — лучшее объяснение

Логарифмы везде.Вы когда-нибудь использовали следующие фразы?

  • 6 цифр
  • Двузначные цифры
  • По порядку величины
  • Процентная ставка

Вы описываете числа в терминах степени 10, логарифма. А процентная ставка — это логарифм роста инвестиций.

Удивлены, что логарифмы так распространены? Я тоже. Большинство попыток Math In the Real World ™ указывают на логарифмы в какой-то загадочной формуле или делают вид, что мы геологи увлечены шкалой Рихтера.«Ученые заботятся о бревнах, и вы должны тоже. Кроме того, можете ли вы представить мир без цинка?»

Нет, нет, нет, нет, нет, нет! (Мама миа!)

Math выражает концепции с помощью таких обозначений, как «ln» или «log». Найти «математику в реальном мире» означает встретить идеи в жизни и увидеть, как можно записать с обозначениями. Не ищите буквальных символов! Когда вы в последний раз писали знак деления? Когда вы в последний раз нарезали еду?

Хорошо, хорошо, мы поняли: что такое логарифмы?

Логарифмы находят причину следствия, т.е.e вход для некоторого выхода

Обычный «эффект» — это рост чего-то, например, переход от \ $ 100 до \ $ 150 за 5 лет. Как это случилось? Мы не уверены, но логарифм находит возможную причину: постоянный возврат ln (150/100) / 5 = 8,1% будет учитывать это изменение. Возможно, это не настоящая причина (весь ли рост произошел в последний год?), Но это плавное среднее значение, которое мы можем сравнить с другими изменениями.

Кстати, понятие «причина и следствие» имеет нюансы.Почему 1000 больше 100?

  • 100 — это 10, которые выросли сами по себе за 2 периода времени (10 $ * 10 $)
  • 1000 — это 10, которые выросли сами по себе за 3 периода времени (10 $ * 10 * 10 $)

Мы можем думать о числах как о выходах (1000 — это «1000 выходов») и входах («Во сколько раз нужно вырасти 10, чтобы получить эти выходы?»). Итак,

  1000 выходов> 100 выходов
  

потому что

  3 входа> 2 входа
  

Или другими словами:

  журнал (1000)> журнал (100)
  

Почему это полезно?

Логарифмы помещают числа в удобную для человека шкалу.80 $ = количество молекул во Вселенной

Шкала от 0 до 80 перевела нас от одного объекта к количеству объектов во Вселенной. Не слишком потрепанный.

Логарифмы считают умножение как шаги

Логарифмы

описывают изменения в терминах умножения: в приведенных выше примерах каждый шаг в 10 раз больше. С натуральным логарифмом каждый шаг в «e» (2,71828 …) раз больше.

Имея дело с серией умножений, логарифмы помогают «подсчитать» их, точно так же, как для нас считается сложение, когда добавляются эффекты.

Покажи мне математику

Время для мяса: посмотрим, где появятся логарифмы!

Шестизначная зарплата или двузначные расходы

Мы описываем числа в терминах их цифр, то есть сколько у них степеней десяти (десятки, сотни, тысячи, десятки тысяч и т. Д.). Добавление цифры означает «умножение на 10», т. Е.

Логарифмы

подсчитывают количество умножений , добавленных к , поэтому, начиная с 1 (одна цифра), мы добавляем еще 5 цифр (10 ^ 5 $) и 100000 получаем результат из 6 цифр.Говорить о «6» вместо «Сто тысяч» — это суть логарифмов. Это дает приблизительное ощущение масштаба, не вдаваясь в детали.

Бонусный вопрос: как бы вы описали 500 000? Сказать «шестизначный» вводит в заблуждение, потому что шестизначное число часто подразумевает что-то близкое к 100000. Подойдет ли «фигура 6,5»?

Не совсем. В нашей голове 6.5 означает «половину» между 6 и 7 цифрами, но это мышление сумматора. В логарифмах «.5» означает половину умножения, т.е..5 $ означает, что квадратный корень из 9 — 3 равен половине умножения, потому что он 1 к 3 и 3 к 9).

Принимая логарифм (500 000), мы получаем 5,7, добавляем 1 для дополнительной цифры, и мы можем сказать, что «500 000 — это 6,7-значное число». Попробуйте здесь:

По порядку величины

Мы, фанаты, любим эту фразу. Это означает примерно «10-кратную разницу», но звучит круче, чем «на 1 цифру больше».

В компьютерах, где все считается битами (1 или 0), каждый бит имеет эффект удвоения (не 10x).16 $ ~ 65 536 раз больше памяти, которую можно адресовать.

Процентные ставки

Как определить темпы роста? Страна не намерена расти на 8,56% в год. Вы смотрите на ВВП в один год и на ВВП в следующий и берете логарифм, чтобы найти неявный темп роста .

Две мои любимые интерпретации натурального логарифма (ln (x)), то есть натурального логарифма 1,5:

  • Предполагая 100% рост, сколько времени вам нужно, чтобы достичь 1.5? (.405, меньше половины периода времени)
  • Если принять 1 единицу времени, как быстро вам нужно вырасти до 1,5? (40,5% в год, непрерывно начисляется)

Логарифмы — это то, как мы определяем, насколько быстро мы растем.

Шкала измерения: Google PageRank

Google присваивает каждой странице в Интернете оценку (PageRank), которая является приблизительной мерой авторитета / важности. Это логарифмическая шкала, которая в моей голове означает «PageRank подсчитывает количество цифр в вашей оценке».4 $ = 10000).

Грубо говоря, у меня бывает около 7000 посещений в день. Используя свои математические вычисления, я могу предположить, что CNN получает около 7000 * 10 000 = 70 миллионов посещений в день. (Как я это сделал? В голове я думаю, что $ 7k * 10k = 70 * k * k = 70 * M $). У них может быть в несколько раз больше (100M, 200M), но, вероятно, не до 700M.

Google передает много информации с очень приблизительной шкалой (1-10).

Шкала измерения: Рихтер, децибел и т. Д.

Вздох. Мы находимся в типичном примере «логарифмов из реального мира»: шкале Рихтера и децибелах.Идея состоит в том, чтобы поместить события, которые могут сильно различаться (землетрясения), в единую шкалу с небольшим диапазоном (обычно от 1 до 10). Как и в случае с PageRank, каждое увеличение на 1 пункт означает 10-кратное повышение мощности. Самое сильное землетрясение, зарегистрированное людьми, произошло 9,5 балла; Удар на полуострове Юкатан, который, вероятно, привел к вымиранию динозавров, составил 13.

Децибелы аналогичны, но могут быть отрицательными. Звуки могут варьироваться от очень тихих (пиндроп) до очень громких (самолет), и наш мозг может все это обработать. На самом деле звук двигателя самолета в миллионы (миллиарды, триллионы) раз мощнее пиндропа, и неудобно иметь шкалу от 1 до миллиарда.Журналы хранят все в разумном масштабе.

Логарифмические графики

Вы часто будете видеть элементы, нанесенные на «логарифмическую шкалу». В моей голове это означает, что одна сторона считает «количество цифр» или «количество умножений», а не само значение. Опять же, это помогает отображать сильно различающиеся события по единой шкале (от 1 до 10, а не от 1 до миллиардов).

Закон

Мура — отличный пример: мы удваиваем количество транзисторов каждые 18 месяцев (изображение любезно предоставлено Википедией).

Изящная особенность графиков в логарифмическом масштабе заключается в том, что экспоненциальные изменения (скорости процессора) отображаются в виде прямой линии. Рост в 10 раз в год означает, что вы неуклонно продвигаетесь по «цифровой» шкале.

Вперед и вверх

Если концепция хорошо известна, но не пользуется большой популярностью, это означает, что нам нужно развивать нашу интуицию. Найдите аналогии, которые работают, и не соглашайтесь на помойку из учебника. В голове:

  • Логарифмы найдите основную причину эффекта (см. Рост, найдите процентную ставку)
  • Они помогают подсчитывать умножения или числа с бонусом в виде частичного подсчета (500k — это 6.0 = 1?)
  • Использование логарифмов в реальном мире
  • Как мыслить экспонентами и логарифмами
  • Различия между дискретным и непрерывным ростом
  • Что на самом деле означает показатель степени?
  • Q: Почему е особенное? (2,718 …, а не 2, 3,7 или другое число?)
  • Что такое логарифм?

    МАТЕМАТИКА
    ОБЗОР: ПОЛЕЗНАЯ МАТЕМАТИКА ДЛЯ ВСЕХ

    РАЗДЕЛ
    4.ЧТО ТАКОЕ ЛОГАРИФМ?


    Логарифм — это
    степень, до которой должно быть возведено число, чтобы получить другое число
    (см. раздел 3 этого обзора математики для получения дополнительной информации.
    о экспонентах). Например, десятичный логарифм 100 равен 2, потому что
    десять в степени двойки равно 100:

    журнал 100 = 2

    потому что

    10 2 =
    100

    Это
    является примером десятичного логарифма.Мы называем это десятичным логарифмом
    потому что десять это число
    это возведено в степень. Базовая единица — это поднимаемое число
    к власти. Есть логарифмы с использованием разных основных единиц. Если
    вы хотели, вы могли бы использовать два в качестве базового блока. Например,
    логарифм восьми по основанию два равен трем, потому что два возведены в
    степень трех равна восьми:

    журнал 2 8
    = 3

    потому что

    2 3 =
    8

    В
    Как правило, вы пишете журнал, за которым следует базовый номер в качестве нижнего индекса.Наиболее распространенные логарифмы:
    логарифмы по основанию 10 и натуральные логарифмы; у них есть специальные обозначения.
    Записывается журнал с основанием десять

    журнал

    и десятичное логарифмическое уравнение
    обычно записывается в виде:

    журнал
    а = г

    Записывается натуральный логарифм.

    пер.

    и натуральное логарифмическое уравнение
    обычно записывается в виде:

    ln a = r

    Итак, когда вы видите журнал
    сам по себе, это означает десятичный журнал.Когда вы видите ln, это означает
    натуральный логарифм (мы определим натуральные логарифмы ниже). В этом
    Конечно, будут использоваться только десятичные и натуральные логарифмы.

    в логарифмах, страница 2


    Для
    подробнее об этом сайте свяжитесь с Distance
    Координатор по образованию.

    Авторские права © 2004 г.
    регентами Миннесотского университета, равные возможности
    работодатель и педагог.

    примеров: решение логарифмических уравнений | Конечная математика

    Решение экспоненциальных уравнений

    1. По возможности выделяйте экспоненциальные выражения
    2. Возьмите логарифм обеих сторон
    3. Используйте свойство экспоненты для логарифмов, чтобы извлечь переменную из экспоненты
    4. Используйте алгебру, чтобы найти переменную.

    Пример 1

    Решите 2
    x = 10 для x .{{x}})}}} = {ln {{({10})}}} [/ латекс] Используя свойство экспоненты для журналов, [латекс] \ displaystyle {x} {ln {{({2})}}} = {ln {{({10})}}} [/ latex] Теперь делим на ln (2), [латекс] \ displaystyle {x} = \ frac {{{ln {{({10})}}}}} {{{ln {{({2})}}}}} примерно {2,861} [/ латекс]

    Обратите внимание, что этот результат совпадает с результатом, который мы получили с помощью формулы изменения базовой.

    Пример 2

    В первом разделе мы предсказали численность населения (в миллиардах) Индии
    t года после 2008 г. с помощью функции f ( t ) 1.{{t}})}}} [/ латекс] Примените свойство экспоненты к правой стороне [латекс] \ displaystyle {ln {{(\ frac {{2}} {{1.14})}}} = {t} {ln {{({1.0134})}}} [/ латекс] Разделите обе стороны на ln (1,0134) [латекс] \ displaystyle {t} = \ frac {{{ln {{(\ frac {{2}} {{1.14}})}}}}} {{{ln {{({1.0134}) }}}}} приблизительно {42.23} {mathtt {{y}}} {mathtt {{e}}} {mathtt {{a}}} {mathtt {{r}}} {mathtt {{s}}} [ / латекс]

    Если этот рост продолжится, модель предсказывает, что население Индии достигнет 2 миллиардов примерно через 42 года после 2008 года или примерно в 2050 году.

    Попробовать 1

    Решить 5 (0,93)
    x = 10.

    Помимо решения экспоненциальных уравнений, логарифмические выражения распространены во многих физических ситуациях.

    Пример 3

    В химии pH — это мера кислотности или основности жидкости. PH связан с концентрацией ионов водорода, [
    H + ], измеренной в молях на литр, по уравнению p H = –log ([ H + ]).

    Если концентрация жидкости составляет 0,0001 моль на литр, определите pH.

    Определите концентрацию ионов водорода в жидкости с pH 7.

    Чтобы ответить на первый вопрос, мы вычисляем выражение –log (0,0001). Хотя мы могли бы использовать для этого наши калькуляторы, на самом деле они нам здесь не нужны, поскольку мы можем использовать обратное свойство журналов:

    –log (0,0001) = –log (10
    –4 ) = — (- 4) = 4

    Чтобы ответить на второй вопрос, нам нужно решить уравнение 7 = –log ([
    H + ]).Начните с выделения логарифма на одной стороне уравнения, умножив обе части на –1: –7 = log ([ H + ])

    Перезапись в экспоненциальную форму дает ответ [
    H + ] = 10 –7 = 0,0000001 моль на литр.

    Логарифмы

    также предоставляют нам механизм для поиска моделей непрерывного роста для экспоненциального роста с учетом двух точек данных.

    Пример 4

    Население вырастает со 100 до 130 за 2 недели.{{{r} {2}}})}}} [/ латекс] [латекс] \ displaystyle {ln {{({1.3})}}} = {2} {r} [/ latex] [латекс] \ displaystyle {r} = \ frac {{{ln {{({1.3})}}}} {{2}} приблизительно {0,1312} [/ latex]

    Это население растет с постоянной скоростью 13,12% в неделю.

    В общем, мы можем связать стандартную форму экспоненты с формой непрерывного роста, отметив (используя
    k для представления скорости непрерывного роста, чтобы избежать путаницы при использовании r двумя разными способами в одной и той же формуле) :

    a (1 + r ) x = ae kx

    (1 +
    r ) x = e kx

    1 +
    r = e k

    Используя это, мы видим, что всегда можно преобразовать из формы непрерывного роста экспоненты в стандартную форму и наоборот.Помните, что скорость непрерывного роста
    k представляет собой номинальную скорость роста без учета эффектов непрерывного начисления процентов, а r представляет собой фактическое увеличение в процентах за одну единицу времени (одна неделя, один год и т. Д.).

    Пример 5

    Продажи компании можно смоделировать с помощью функции
    S ( t ) = 5000 e 0,12 t , при этом t измеряются в годах. Найдите годовой темп роста.

    Отмечая, что 1 +
    r = e k , затем r = e 0,12 — 1 = 0,1275, поэтому годовой темп роста составляет 12,75%. Функцию продаж также можно записать в виде S ( t ) = 5000 (1 + 0,1275) t .

    • Решение экспоненциальных уравнений

    Попробовать сейчас Ответы

    [латекс] \ displaystyle \ frac {{{ln {{({2})}}}}} {{ln {{({0.93})}}}} примерно — {9.5513} [/ латекс]


    Дэвид Липпман и Мелони Расмуссен, Книжный магазин Open Text, Precalculus: An Investigation of Functions, «Глава 4: Экспоненциальные и логарифмические функции», под лицензией CC BY-SA 3.0.

    BioMath: логарифмические функции

    Логарифмы встречаются во всех биологических науках. Несколько примеров
    включают расчет pH раствора или изменение свободной энергии
    связаны с биохимическими реакциями.Чтобы понять, как решить эти
    уравнения, мы должны сначала рассмотреть определение логарифма.

    Определение — Формальное определение логарифма выглядит следующим образом:

    Основание логарифм
    положительного числа x — это показатель степени, который вы
    получить, когда вы напишете x как
    степень a , где a > 0 и a ≠ 1 .
    То есть

    журнал a x = k , если
    и только если a k = x.

    Ключ к взятию
    логарифм x > 0 — это переписать x , используя
    база а . Например,

    журнал 2 32 = 5

    можно переписать как

    2 5 = 32,

    Кто такое придумал?

    Джон Напье, шотландский математик, приписывают изобретение логарифмов.Его книга « Описание чудесного закона логарифмов » была опубликована в 1614 году. Напье разработал метод, упрощающий вычисления, используя сложение и вычитание, а не умножение и деление. Сегодня мы обычно используем логарифмы по основанию 10, общие логарифмы или логарифмы по основанию e, или натуральные логарифмы. В публикации Нэпьера он описывает логи по основанию 2.

    Некоторые примеры логарифмов

    Логарифмы, как и показатели степени, могут иметь разное основание.В биологическом
    наук, вы, вероятно, столкнетесь с логарифмом по основанию 10, известным как
    десятичный логарифм и обозначается просто как журнал; и логарифм по основанию и ,
    известен как натуральный журнал и обозначается как ln. Большинство калькуляторов легко
    вычислить эти широко используемые логарифмы.

    B ase 10 логарифм

    Десятичный логарифм положительного числа x ,
    это показатель степени, который вы получаете, когда
    вы пишете x как степень
    10.Это,

    журнал x = k , если
    и только если 10 k = x

    Вычислить десятичный логарифм x> 0 вручную можно только при
    особые обстоятельства,
    и мы сначала рассмотрим их. Начнем с вычислений
    стоимость,

    журнал 10.

    Согласно нашему определению десятичного логарифма, нам нужно переписать
    x = 10 с использованием базы
    10.Это легко сделать, потому что 10 = 10 1 . Таким образом, показатель степени k , мы получаем
    при переписывании 10 с использованием
    база 10 равна, k = 1. Таким образом, мы заключаем, что

    журнал 10 = журнал 10 1 = 1.

    Хотя этот пример довольно прост, это хорошая практика.
    следовать этому методу решения. Сейчас
    попробуйте следующие упражнения.

    Проверьте себя с помощью следующих упражнений

    По мере того, как вы работали над этими упражнениями, заметили ли вы, что выход логарифмов увеличивается линейно, по мере экспоненциального увеличения входных значений ?

    Натуральные логарифмы

    Натуральный логарифм положительного числа x ,
    это показатель степени, который вы получаете, когда пишете x как
    мощность e .Напомним, что

    журнал e x =
    ln x

    следовательно

    ln x = k если
    и только если e k = x .

    Логарифмические вычисления, которые нельзя выполнить вручную.

    Теперь предположим, что вас попросили вычислить значение log 20 . Что бы вы
    сделать (или попытаться сделать), чтобы получить ответ? Вы замечаете что-нибудь другое в
    Эта проблема?

    Как вы, скорее всего, заметили, не существует целого числа k , такого что 10 k =
    20. Итак, в этом случае вам нужно будет положиться на свой калькулятор.
    Используя свой калькулятор, вы найдете:

    журнал 20 ≈ 1.30.

    Помните, что это правда, потому что

    10 1,30 ≈ 20,

    Опять проверь себя

    После выполнения этих упражнений вы заметите, что ваш
    ответы (выходы) малы по сравнению с вашими большими входами. Помнить
    что логарифмы преобразуют экспоненциально возрастающие входные данные в
    линейно возрастающие выходы.Это очень удобно для биологов.
    которые работают над многими порядками и над множеством разных
    напольные весы.

    Обратные

    Поскольку экспоненциальная и логарифмическая функции являются обратными,
    область логарифмов — это диапазон
    экспонент (т.е. положительных действительных чисел), а диапазон
    логарифмов — это область
    экспоненты (т.е. все действительные числа). Это верно для всех логарифмов,
    вне зависимости от базы.

    Отзыв
    что экспоненциальная функция с основанием a
    записывается как f ( x ) = a x . Обратное к этому
    функция — это основание , логарифмическая функция , записанная как,

    f −1 ( x )
    = г ( x ) = бревно a x .

    Если нет явного написания нижнего индекса a ,
    предполагается, что логарифм является общим (т. е.
    основание 10). Из этого обозначения есть одно специальное исключение.
    для основания e ≈ 2,718, что называется натуральным логарифмом ,

    г ( x ) = бревно e x = ln x .

    Для вычисления основания и логарифма x> 0 ,
    перепишите x , используя
    base a (так же, как и для
    основание 10). Например, предположим, что a =
    2 и мы хотим вычислить,

    журнал 2 8.

    Чтобы найти это значение вручную, мы преобразуем число 8, используя
    база 2 как,

    журнал 2 8 = журнал 2 2 3 = 3,

    , как и для базы 10.

    Используя эту методологию, проверьте себя
    путем вычисления следующего вручную.

    *****

    В следующем разделе мы опишем свойства логарифмов.

    Добавить комментарий

    Ваш адрес email не будет опубликован. Обязательные поля помечены *

    2024 © Все права защищены.